Business A Practical Introduction By Williams – Test Bank A+

$35.00
Business A Practical Introduction By Williams – Test Bank A+

Business A Practical Introduction By Williams – Test Bank A+

$35.00
Business A Practical Introduction By Williams – Test Bank A+

1) The primary motive of an enterprise is to make a profit.

Answer: TRUE

Explanation: The mission of any business is to generate wealth, or earn a profit.

Page Ref: 164

Learning Outcome: Describe the role of small businesses in the economy

Difficulty: Easy

Objective: 6.1 Do I have what it takes to be a small-business entrepreneur?

Classification: Concept

2) A skunkworks is a group of people who split off from a company to use its proprietary technology and trade secrets for starting their own enterprise.

Answer: FALSE

Explanation: A skunkworks is a team that is separated from the normal operation of an organization and asked to produce a new, innovative project for that organization.

Page Ref: 166

Difficulty: Easy

Objective: 6.1 Do I have what it takes to be a small-business entrepreneur?

Classification: Concept

3) More entrepreneurs are driven by opportunity than necessity.

Answer: TRUE

Explanation: Most entrepreneurs fall under the category of opportunity entrepreneurs; they are ambitious and start their own businesses in voluntary pursuit of opportunity.

Page Ref: 166

Difficulty: Easy

Objective: 6.1 Do I have what it takes to be a small-business entrepreneur?

Classification: Concept

4) A small business that requires little investment to become operational is called an ultralight startup.

Answer: TRUE

Explanation: Many ultralights are Web-based businesses.

Page Ref: 171

Difficulty: Easy

Objective: 6.2 How important is small business, and what newer types might interest me?

Classification: Concept

5) After working hard and facing enormous risks, many entrepreneurs sell their business to realize a substantial profit.

Answer: TRUE

Explanation: Some people develop businesses with the intention of creating a valuable enterprise that can be sold for a healthy return on their investment. Creating wealth is a primary driver for business.

Page Ref: 173

Difficulty: Easy

Objective: 6.2 How important is small business, and what newer types might interest me?

Classification: Concept

6) A service corps consists of individuals and companies in a specific business or industry organized to promote common interests.

Answer: FALSE

Explanation: The term for this type of group is a trade association.

Page Ref: 176

Difficulty: Easy

Objective: 6.3 If I wanted to start a company, how should I go about it?

Classification: Concept

7) A business model defines the need a proposed business will fill as well as its operations, components, functions, and expected revenues and expenses.

Answer: TRUE

Explanation: Clarifying the business model is an important step before writing a business plan for a new enterprise.

Page Ref: 177

Learning Outcome: Explain the entrepreneurship process

Difficulty: Easy

Objective: 6.3 If I wanted to start a company, how should I go about it?

Classification: Concept

8) The Small Business Administration offers a variety of direct loans to help new businesses get started.

Answer: FALSE

Explanation: The SBA does not make direct loans; it guarantees business loans made by banks.

Page Ref: 181

Difficulty: Easy

Objective: 6.4 What are various ways of financing my new business?

Classification: Concept

9) A venture capitalist receives an ownership stake in a new business in return for its investment.

Answer: TRUE

Explanation: Venture capitalists are generally companies, not individuals. They provide little financing for small businesses, focusing on larger enterprises in several industry sectors.

Page Ref: 182

Difficulty: Easy

Objective: 6.4 What are various ways of financing my new business?

Classification: Concept

10) Tax credits are a key tool that governments use to encourage business development in enterprise zones.

Answer: TRUE

Explanation: An enterprise zone is a specific geographic area in which government tries to attract business investment by offering lower taxes and other government support.

Page Ref: 182

Difficulty: Easy

Objective: 6.4 What are various ways of financing my new business?

Classification: Concept

11) The term going public refers to the launch of a company’s website.

Answer: FALSE

Explanation: “Going public” means that a privately owned company becomes a publicly owned company through stock issued for sale to the public.

Page Ref: 182

Difficulty: Easy

Objective: 6.4 What are various ways of financing my new business?

Classification: Concept

12) Big businesses are more likely to fail than small businesses.

Answer: FALSE

Explanation: For reasons ranging from lack of financial support to aggressive competition, small businesses are more apt to fail than big businesses are.

Page Ref: 183

Difficulty: Easy

Objective: 6.5 How can I avoid failure and keep my business healthy?

Classification: Concept

13) Small businesses can fail due to problems with economies of scale.

Answer: TRUE

Explanation: Economies of scale refer to savings realized from buying materials or manufacturing products in large quantities. Small businesses often don’t have the resources or market to benefit from this dynamic.

Page Ref: 183

Difficulty: Easy

Objective: 6.5 How can I avoid failure and keep my business healthy?

Classification: Concept

14) The incoming and outgoing of money in a business enterprise is called total balance of payments.

Answer: FALSE

Explanation: Incoming and outgoing of money in a business is called cash flow.

Page Ref: 183

Difficulty: Easy

Objective: 6.5 How can I avoid failure and keep my business healthy?

Classification: Concept

15) Which of the following is NOT a U.S. government criterion used to define a small business?

  1. A) ownership
  2. B) stature in field of operation
  3. C) facility location
  4. D) number of employees
  5. E) sales revenue

Answer: C

Explanation: C) The Small Business Administration does not use the location of a business to determine its status. Although many small businesses are home-based operations, many others are not.

Page Ref: 163

Learning Outcome: Describe the role of small businesses in the economy

Difficulty: Moderate

Objective: 6.1 Do I have what it takes to be a small-business entrepreneur?

Classification: Concept

16) Which of the following would be LEAST interested in building a high-growth business?

  1. A) entrepreneurial team
  2. B) necessity entrepreneur
  3. C) classic entrepreneur
  4. D) opportunity entrepreneur
  5. E) micropreneur

Answer: E

Explanation: E) A micropreneur usually has a small operation, often a home-based business, as a means to earn a living and have a balanced lifestyle.

Page Ref: 163

Learning Outcome: Explain the entrepreneurship process

Difficulty: Moderate

Objective: 6.1 Do I have what it takes to be a small-business entrepreneur?

Classification: Concept

17) Which of the following terms is most often associated with home-based businesses?

  1. A) classic entrepreneur
  2. B) micropreneur
  3. C) necessity entrepreneur
  4. D) opportunity entrepreneur
  5. E) intrapreneur

Answer: B

Explanation: B) Micropreneurs start businesses that remain small, let them do the kind of work they want to do, and offer a balanced lifestyle. Micropreneur businesses are often home-based.

Page Ref: 163

Learning Outcome: Explain the entrepreneurship process

Difficulty: Moderate

Objective: 6.1 Do I have what it takes to be a small-business entrepreneur?

Classification: Concept

18) Which of the following would NOT be considered a small business by the U.S. government?

  1. A) a beauty shop owned by two women, with monthly revenue of $23,000
  2. B) a sandwich shop owned by a corporation, with monthly revenue of $34,000
  3. C) a food truck owned by a college student, with monthly revenue of $6,000
  4. D) a pool cleaning service owned by a family, with monthly revenue of $160,000
  5. E) a luxury online gift service owned by a retired woman, with monthly revenue of $225,000

Answer: B

Explanation: B) An operation must be independently owned and operated to be defined as a small business by the U.S. government.

Page Ref: 163

Difficulty: Moderate

Objective: 6.1 Do I have what it takes to be a small-business entrepreneur?

Classification: Application

19) What is the primary motive of an enterprise?

  1. A) to sell products and services
  2. B) to make a profit
  3. C) to generate revenue
  4. D) to create jobs
  5. E) to enhance the community

Answer: B

Explanation: B) An enterprise is defined as an endeavor in which the primary motive is to make a profit.

Page Ref: 164

Difficulty: Moderate

Objective: 6.1 Do I have what it takes to be a small-business entrepreneur?

Classification: Concept

20) Business always involves some level of risk. Which of the following is subject to the greatest individual risk?

  1. A) micropreneur
  2. B) classic entrepreneur
  3. C) skunkworks member
  4. D) entrepreneurial team member
  5. E) intrapreneur

Answer: B

Explanation: B) A classic entrepreneur is an individual who starts a business to pursue an opportunity. Micropreneurs operate on a smaller scale and seek a balanced lifestyle. Intrapreneurs have the safety of an existing job. Risk is shared across the members of skunkworks and entrepreneurial teams.

Page Ref: 165

Difficulty: Moderate

Objective: 6.1 Do I have what it takes to be a small-business entrepreneur?

Classification: Concept

21) What is the term for a person who works within an organization to develop an opportunity for a new product or service?

  1. A) opportunity entrepreneur
  2. B) classic entrepreneur
  3. C) necessity entrepreneur
  4. D) intrapreneur
  5. E) resident entrepreneur

Answer: D

Explanation: D) An intrapreneur mobilizes the organization’s resources to realize an opportunity for a new product or service.

Page Ref: 165

Learning Outcome: Explain the entrepreneurship process

Difficulty: Moderate

Objective: 6.1 Do I have what it takes to be a small-business entrepreneur?

Classification: Concept

22) Nathan has been working with eight other colleagues on a special project to develop a groundbreaking product for consumer medical technology. The employees were selected by the company’s CEO and have been given a secure floor in the facility and a hefty R&D budget. What term best describes this group?

  1. A) opportunity entrepreneurs
  2. B) entrepreneurial team
  3. C) micropreneurs
  4. D) skunkworks
  5. E) intrapreneurs

Answer: D

Explanation: D) These employees do make up an entrepreneurial team and may be considered individual intrapreneurs, but being formally separated from normal operations to develop an innovative product qualifies the team as a skunkworks.

Page Ref: 166

Difficulty: Easy

Objective: 6.1 Do I have what it takes to be a small-business entrepreneur?

Classification: Application

23) Sandra owns an educational technology company. She feels the field is getting crowded and wants to develop an innovative software product that will help the company stay ahead of the pack. What is her best strategy to accomplish this goal?

  1. A) Hire a group of new employees as an entrepreneurial team.
  2. B) Hire a classic entrepreneur to serve as a consultant.
  3. C) Choose her best, most creative employees for a skunkworks.
  4. D) Post internally for an intrapreneur position.
  5. E) Outsource research and development to a micropreneur.

Answer: C

Explanation: C) A skunkworks is a group of employees separated from normal company operations to work on developing a new product. Sandra will benefit from the employees’ familiarity with the company as well as their skills.

Page Ref: 166

Difficulty: Moderate

Objective: 6.1 Do I have what it takes to be a small-business entrepreneur?

Classification: Application

24) Johnetta was laid off from her job as a receptionist two years ago. After applying unsuccessfully for hundreds of jobs, she decided to take a chance selling her berry pies. She earns a good living and is quite happy with her little slice of the American dream. Which term best describes Johnetta?

  1. A) classic entrepreneur
  2. B) intrapreneur
  3. C) opportunity entrepreneur
  4. D) necessity entrepreneur
  5. E) transitional entrepreneur

Answer: D

Explanation: D) Johnetta started her pie business in an effort to replace lost income after being out of work for a long period.

Page Ref: 166

Difficulty: Moderate

Objective: 6.1 Do I have what it takes to be a small-business entrepreneur?

Classification: Application

25) Which of the following operates in “crisis mode” when starting an enterprise?

  1. A) intrapreneur
  2. B) necessity entrepreneur
  3. C) opportunity entrepreneur
  4. D) classic entrepreneur
  5. E) entrepreneurial team

Answer: B

Explanation: B) Necessity entrepreneurs are people who suddenly must earn a living and are trying to replace lost income.

Page Ref: 166

Difficulty: Moderate

Objective: 6.1 Do I have what it takes to be a small-business entrepreneur?

Classification: Concept

26) Which of the following is NOT a common psychological characteristic of entrepreneurs?

  1. A) high energy level
  2. B) high need for achievement
  3. C) high need for delegating control
  4. D) high tolerance for ambiguity
  5. E) high self-confidence

Answer: C

Explanation: C) Entrepreneurs tend to have a high need for personal control over their own destinies.

Page Ref: 167

Learning Outcome: Explain the entrepreneurship process

Difficulty: Moderate

Objective: 6.1 Do I have what it takes to be a small-business entrepreneur?

Classification: Concept

27) Which of the following terms is best associated with being an entrepreneur?

  1. A) safety
  2. B) certainty
  3. C) predictability
  4. D) ambiguity
  5. E) reliability

Answer: D

Explanation: D) The business world is filled with change and risk; entrepreneurs must have a high tolerance for ambiguity.

Page Ref: 167

Learning Outcome: Explain the entrepreneurship process

Difficulty: Moderate

Objective: 6.1 Do I have what it takes to be a small-business entrepreneur?

Classification: Concept

28) After 25 years of sitting at a desk from 8:30 to 5:00 and tolerating some inept bosses, Leo took his savings and opened a flower shop. He decides when to open and what to sell, and has never been happier. Which of the common entrepreneurial characteristics does Leo’s story best describe?

  1. A) high self-confidence
  2. B) high tolerance for risk
  3. C) high action orientation
  4. D) high need for personal control
  5. E) high energy level

Answer: D

Explanation: D) Entrepreneurs like to have a feeling of personal control over their destinies. Leo now has this in running his own business.

Page Ref: 167

Learning Outcome: Explain the entrepreneurship process

Difficulty: Easy

Objective: 6.1 Do I have what it takes to be a small-business entrepreneur?

Classification: Application

29) Nina is an overachiever who is always on the go. She left her job as a production manager at a successful company because she disliked the owner’s go-with-the-flow approach and preferred detailed plans. Based on this profile, what does Nina need to develop to be a successful entrepreneur?

  1. A) high energy level
  2. B) high self-confidence
  3. C) high tolerance for ambiguity
  4. D) high need for achievement
  5. E) high belief in personal control

Answer: C

Explanation: C) Managers and entrepreneurs need to be able to make decisions based on incomplete information. Entrepreneurs are also trying things they haven’t done before.

Page Ref: 167

Difficulty: Moderate

Objective: 6.1 Do I have what it takes to be a small-business entrepreneur?

Classification: Application

30) Why it is important for an entrepreneur to have high self-confidence?

  1. A) Confident people tend to have more money.
  2. B) Confidence is needed to act decisively.
  3. C) An entrepreneur needs to be able to succeed at any business.
  4. D) Entrepreneurs need motivation.
  5. E) Entrepreneurs need to be hardworking.

Answer: B

Explanation: B) Many traits are necessary for an entrepreneur to be successful. Specifically, an entrepreneur needs to have confidence in order to act decisively.

Page Ref: 167

Learning Outcome: Explain the entrepreneurship process

Difficulty: Moderate

Objective: 6.1 Do I have what it takes to be a small-business entrepreneur?

Classification: Concept

31) Why does working at home pose special challenges for entrepreneurs?

  1. A) There are additional costs involved.
  2. B) It can make it hard to find customers.
  3. C) Small companies do not succeed as well as larger ones.
  4. D) Steps have to be taken to keep work and personal life separate.
  5. E) Entrepreneurship is best done within the context of a larger company.

Answer: D

Explanation: D) Entrepreneurs who work from home have the challenge of keeping work and personal life separate, since both take place at the same location.

Page Ref: 170

Learning Outcome: Describe the role of small businesses in the economy

Difficulty: Moderate

Objective: 6.2 How important is small business, and what newer types might interest me?

Classification: Concept

32) Small companies with fewer than 500 employees account for what proportion of U.S. businesses?

  1. A) about 35 percent
  2. B) about 50 percent
  3. C) about 75 percent
  4. D) about 85 percent
  5. E) over 95 percent

Answer: E

Explanation: E) Small companies with fewer than 500 employees account for 99.7 percent of businesses in the United States.

Page Ref: 168

Difficulty: Moderate

Objective: 6.2 How important is small business, and what newer types might interest me?

Classification: Concept

33) Which of the following statements does NOT support the common notion that small business is the backbone of America?

  1. A) Small businesses provide the first job for most entrants to the labor force.
  2. B) Companies with fewer than 500 employees comprise over 99 percent of all U.S. businesses.
  3. C) Small firms are twice as innovative per employee than large firms.
  4. D) Small businesses are more apt to fail than big businesses are.
  5. E) Small businesses have generated 64 percent of net new jobs annually for 15 years.

Answer: D

Explanation: D) Small businesses provide numerous benefits to the economy. They also face higher levels of risk.

Page Ref: 168

Difficulty: Moderate

Objective: 6.2 How important is small business, and what newer types might interest me?

Classification: Synthesis

34) Compared to big businesses, small businesses contribute to the overall economic vitality of the United States in all of the following ways EXCEPT which?

  1. A) promote more innovation
  2. B) provide more first jobs
  3. C) enjoy more economies of scale
  4. D) employ more workers
  5. E) generate more new jobs

Answer: C

Explanation: C) Small business are twice as innovative per employee as larger firms, provide the first job for most entrants to the labor force, employ over half of U.S. workers, and generated 64 percent of net new jobs annually over the 15 years ending in 2008. Compared to big businesses, however, small businesses enjoy unfavorable economies of scale.

Page Ref: 168

Learning Outcome: Describe the role of small businesses in the economy

Difficulty: Moderate

Objective: 6.2 How important is small business, and what newer types might interest me?

Classification: Synthesis

35) Which of the following does NOT describe many home-based businesses in the United States?

  1. A) 1- or 2-person operations
  2. B) high use of Web technology
  3. C) primary emphasis on product sales
  4. D) financed for less than $10,000
  5. E) comprise one-half of new small businesses

Answer: C

Explanation: C) Many home-based businesses offer services ranging from software development to transcription and tutoring.

Page Ref: 169

Difficulty: Moderate

Objective: 6.2 How important is small business, and what newer types might interest me?

Classification: Concept

36) Working at a home-based business can be challenging. Which of the following is NOT a common strategy to be successful operating a business at home?

  1. A) stick to a work routine
  2. B) network with other professionals
  3. C) dress professionally
  4. D) outsource tedious tasks
  5. E) dedicate a computer for work

Answer: D

Explanation: D) Not every aspect of running a business is pleasant; home-based business owners often have to wear many hats.

Page Ref: 170

Difficulty: Moderate

Objective: 6.2 How important is small business, and what newer types might interest me?

Classification: Concept

37) Joan set up shop as a virtual concierge for traveling executives. She gets e-mail requests from clients on her home computer before they fly in to her city, and she makes sure they have everything they need upon arrival. The work is a perfect fit and she’s completely content. Which term best describes Joan?

  1. A) skunkworker
  2. B) bedouin
  3. C) freelancer
  4. D) micropreneur
  5. E) opportunity entrepreneur

Answer: D

Explanation: D) Joan has a small, home-based business with no plans for significant growth.

Page Ref: 170

Difficulty: Moderate

Objective: 6.2 How important is small business, and what newer types might interest me?

Classification: Application

38) Ramon believed he could earn money helping students with homework online. He built a Homework SOS website and paid the business filing fee and site host. All revenue comes from memberships. Within a few months, Ramon was making $1,000 a month in profit. Which term best describes the business?

  1. A) freelance business
  2. B) home-based business
  3. C) ultralight startup
  4. D) affiliated marketer
  5. E) bedouin operation

Answer: C

Explanation: C) Homework SOS is a Web-based business that was started with very little initial financial investment.

Page Ref: 171

Difficulty: Moderate

Objective: 6.2 How important is small business, and what newer types might interest me?

Classification: Application

39) What is the primary distinguishing characteristic of an ultralight startup?

  1. A) fewer than five employees
  2. B) low initial investment
  3. C) home-based operation
  4. D) Web-based operation
  5. E) bedouin management

Answer: B

Explanation: B) Ultralight startups require very little initial capital and can help owners recoup their investment very quickly.

Page Ref: 171

Difficulty: Moderate

Objective: 6.2 How important is small business, and what newer types might interest me?

Classification: Concept

40) Which of the following would be LEAST likely to create an ultralight startup with the goal of selling the company for a substantial profit within five years?

  1. A) intrapreneur
  2. B) entrepreneurial team
  3. C) classic entrepreneur
  4. D) micropreneur
  5. E) opportunity entrepreneur

Answer: D

Explanation: D) Micropreneurs tend to be small, home-based operations created by people who want to work for themselves and have a balanced lifestyle.

Page Ref: 173

Difficulty: Moderate

Objective: 6.2 How important is small business, and what newer types might interest me?

Classification: Synthesis

41) Tyrell has always wanted to own a car repair shop. Rather than start from scratch, he’s looking to buy an existing business. To make a smart purchase, Tyrell wants to know everything about how the shop he is looking at operates. Which of the following would NOT be part of his research on business operations?

  1. A) services offered
  2. B) customer base
  3. C) employees
  4. D) area competition
  5. E) business price

Answer: E

Explanation: E) The sale price of the shop is not part of the business operations. However, after thoroughly examining the business operations, Tyrell will know whether the sale price is appropriate.

Page Ref: 173

Difficulty: Moderate

Objective: 6.2 How important is small business, and what newer types might interest me?

Classification: Application

42) Buying an existing business can help to minimize what core challenge facing every business?

  1. A) retaining customers
  2. B) risk
  3. C) financing
  4. D) marketing
  5. E) quality control

Answer: B

Explanation: B) Every business faces some risk and uncertainty. In buying an existing operation, the previous owner has usually overcome most of the risks.

Page Ref: 173

Difficulty: Moderate

Objective: 6.2 How important is small business, and what newer types might interest me?

Classification: Concept

43) After 25 years of running a successful hair salon, Connie wants to sell the business and buy a retirement house in Arizona. What is the first thing she should do to implement her plan?

  1. A) begin networking to find buyers
  2. B) hire a business lawyer
  3. C) post the business on eBay
  4. D) ask employees if they want to buy her out
  5. E) research prices for similar businesses

Answer: B

Explanation: B) Selling a business is a complex process. Hiring an attorney—and other professional advisors such as an accountant—will ensure that Connie gets fair value for all the time, effort, and resources she’s put into her salon.

Page Ref: 173

Difficulty: Moderate

Objective: 6.2 How important is small business, and what newer types might interest me?

Classification: Application

44) The Dyson brand of vacuum cleaners emphasizes one key selling point in its advertisements: the product will never lose suction power over time, as other vacuums do. This reflects which of the following approaches to coming up with a business idea?

  1. A) adapt a product from one market to another market
  2. B) simplify an existing product
  3. C) spin one feature off from a multifeatured product
  4. D) enhance an existing product
  5. E) make a product less expensive

Answer: D

Explanation: D) Maintaining consistent suction over time is an enhancement to an existing product—and a good selling point.

Page Ref: 174

Difficulty: Moderate

Objective: 6.3 If I wanted to start a company, how should I go about it?

Classification: Application

45) When someone has a brilliant idea for a new business, what is the first thing he or she should do to get started on the right track?

  1. A) ask friends their opinion of the idea
  2. B) write a business plan
  3. C) create a logo and website
  4. D) learn about the industry
  5. E) identify potential funders

Answer: D

Explanation: D) Rather than jump right out of the gate, developing a deeper understanding of the proposed business through research, training, and work experience can answer a lot of initial questions and set the stage for the next steps, such as a business plan.

Page Ref: 176

Learning Outcome: Explain the entrepreneurship process

Difficulty: Moderate

Objective: 6.3 If I wanted to start a company, how should I go about it?

Classification: Concept

46) Which of the following terms best describes the role of a mentor in starting a business?

  1. A) consultant
  2. B) coach
  3. C) employee
  4. D) investor
  5. E) trainer

Answer: B

Explanation: B) A mentor is an experienced person who coaches and guides lesser-experienced people by helping them understand an industry culture or structure. Because a mentor is a strong advocate and source of support, “coach” is the most appropriate term.

Page Ref: 176

Difficulty: Moderate

Objective: 6.3 If I wanted to start a company, how should I go about it?

Classification: Concept

47) Which of the following is the best source for research data, certification and training, and professional contacts for an entrepreneur who wants to learn more about a particular industry?

  1. A) Service Corps of Retired Executives (SCORE)
  2. B) the Web
  3. C) mentors
  4. D) trade associations
  5. E) Active Corps of Executives (ACE)

Answer: D

Explanation: D) Trade associations consist of individuals and companies in a specific industry organized to promote common interests. They can provide research data, training, certification, and links to others in the industry.

Page Ref: 176

Learning Outcome: Explain the entrepreneurship process

Difficulty: Moderate

Objective: 6.3 If I wanted to start a company, how should I go about it?

Classification: Concept

48) Which of the following is NOT a rationale for creating a business plan?

  1. A) to outline the goals of a proposed company
  2. B) to secure funding for a proposed company
  3. C) to lay out methods to achieve the proposed company’s goals
  4. D) to clarify the business model for the proposed company
  5. E) to state standards for measuring success of the proposed company

Answer: D

Explanation: D) The business model is defined and clarified prior to developing the business plan. It defines the basic idea behind the proposed business.

Page Ref: 177

Learning Outcome: Explain the entrepreneurship process

Difficulty: Moderate

Objective: 6.3 If I wanted to start a company, how should I go about it?

Classification: Concept

49) Which of the following commonly develops a business idea within an already established business model?

  1. A) micropreneur
  2. B) classic entrepreneur
  3. C) necessity entrepreneur
  4. D) intrapreneur
  5. E) opportunity entrepreneur

Answer: D

Explanation: D) An intrapreneur works within an existing company operating under a business model and mobilizes its resources to develop new products or services. By comparison, the other types of entrepreneurs start from scratch.

Page Ref: 177

Difficulty: Moderate

Objective: 6.3 If I wanted to start a company, how should I go about it?

Classification: Concept

50) Which of the following has the LEAST need for creating a complex business plan?

  1. A) opportunity entrepreneur
  2. B) classic entrepreneur
  3. C) micropreneur
  4. D) necessity entrepreneur
  5. E) entrepreneurial team

Answer: C

Explanation: C) Micropreneurs tend to operate home-based businesses to make a living and have a balanced lifestyle. The scale of their operation generally does not warrant a detailed business plan. But with larger goals, even a micropreneur can benefit from creating a plan.

Page Ref: 177

Learning Outcome: Explain the entrepreneurship process

Difficulty: Moderate

Objective: 6.3 If I wanted to start a company, how should I go about it?

Classification: Synthesis

51) What is the most important information that potential investors review to determine whether to provide capital for a new business?

  1. A) business model
  2. B) owner resume
  3. C) business website
  4. D) business plan
  5. E) owner certifications

Answer: D

Explanation: D) A business plan outlines a company’s goals, methods for achieving them, and standards for measuring success. The more money required of investors, the more comprehensive the business plan should be.

Page Ref: 177

Learning Outcome: Explain the entrepreneurship process

Difficulty: Moderate

Objective: 6.3 If I wanted to start a company, how should I go about it?

Classification: Concept

52) Jason wants to start a t-shirt printing business, but he needs initial capital. He attends networking functions for bankers and investors and chats up his idea, but he gets no takers. What would be the best strategy for Jason to get funding for his business concept?

  1. A) meet with a mentor
  2. B) develop a business plan
  3. C) work part time in a screenprinting shop
  4. D) join a trade association
  5. E) get trained in new printing technology

Answer: B

Explanation: B) Creating a business plan provides potential funders with the details they need to make an informed decision about whether a business idea is a good investment.

Page Ref: 177

Learning Outcome: Explain the entrepreneurship process

Difficulty: Easy

Objective: 6.3 If I wanted to start a company, how should I go about it?

Classification: Application

53) Which of the following is NOT part of the business model for a proposed enterprise?

  1. A) the business operations
  2. B) anticipated revenues and expenses
  3. C) business components and functions
  4. D) market need to be filled
  5. E) qualifications of the owner

Answer: E

Explanation: E) A business model defines the basic idea behind a business. An owner’s qualifications should be considered before writing a business plan but are not part of the business model.

Page Ref: 177

Learning Outcome: Explain the entrepreneurship process

Difficulty: Moderate

Objective: 6.3 If I wanted to start a company, how should I go about it?

Classification: Concept

54) Which of the following funding sources is LEAST likely to require a business plan to make a decision on investing in a proposed business?

  1. A) venture capitalists
  2. B) banks and credit unions
  3. C) friends and associates
  4. D) angel investors
  5. E) peer-to-peer lending groups

Answer: C

Explanation: C) Friends and associates are considered a more informal investor group than the other options. However, to protect their investment—and the relationship—friends and associates may want to review a business plan as well.

Page Ref: 177

Learning Outcome: Explain the entrepreneurship process

Difficulty: Moderate

Objective: 6.3 If I wanted to start a company, how should I go about it?

Classification: Synthesis

55) Tuan is eager to get his green toy manufacturing business started. But he is a very deliberate person and wants to be sure his strategy is sound. What is Tuan’s best option to accomplish this before going forward?

  1. A) join a toy manufacturing trade association
  2. B) map out his business model
  3. C) meet with a SCORE member
  4. D) prepare a business plan
  5. E) meet regularly with a mentor

Answer: D

Explanation: D) All of the options will be beneficial, but the level of detail and thinking reflected in a business plan will help Tuan thoroughly assess and refine his business strategy.

Page Ref: 177

Learning Outcome: Explain the entrepreneurship process

Difficulty: Moderate

Objective: 6.3 If I wanted to start a company, how should I go about it?

Classification: Application

56) The comprehensive analysis required in preparing a business plan can help an entrepreneur identify what critical factor related to success or failure in business?

  1. A) keeping good records
  2. B) adequate management skills
  3. C) credit
  4. D) government paperwork
  5. E) hiring good employees

Answer: B

Explanation: B) Because a business plan requires detailed thinking about a business model, marketing, legal and financial issues, and management, preparing a plan can help entrepreneurs to recognize their own shortcomings and either enhance their skills or ask for assistance.

Page Ref: 177

Difficulty: Moderate

Objective: 6.3 If I wanted to start a company, how should I go about it?

Classification: Synthesis

57) Which of the following is NOT covered in the business plan for a proposed service company?

  1. A) legal plan
  2. B) marketing plan
  3. C) management team
  4. D) manufacturing plan
  5. E) financing plan

Answer: D

Explanation: D) Because a company that sells services does not create any tangible products, information on manufacturing is not relevant in a plan for this type of business.

Page Ref: 178

Difficulty: Moderate

Objective: 6.3 If I wanted to start a company, how should I go about it?

Classification: Concept

58) What do mentors mainly provide to someone starting a small business?

  1. A) investment of capital
  2. B) grants for startup costs
  3. C) experienced advice
  4. D) additional employees
  5. E) legal counsel

Answer: C

Explanation: C) Mentors are experienced businesspeople who can help give good advice to someone starting up a business for the first time.

Page Ref: 176

Learning Outcome: Explain the entrepreneurship process

Difficulty: Moderate

Objective: 6.3 If I wanted to start a company, how should I go about it?

Classification: Concept

59) Which of the following plays a role in funding the vast majority of startup businesses in the United States?

  1. A) peer-to-peer lenders
  2. B) angel investors
  3. C) family and friends
  4. D) venture capitalists
  5. E) the Small Business Administration

Answer: C

Explanation: C) Family and friends provide funding for 90 percent of startup businesses.

Page Ref: 180

Difficulty: Moderate

Objective: 6.4 What are various ways of financing my new business?

Classification: Concept

60) Which source of financing often requires a business owner to put up a substantial amount of his or her personal funds to demonstrate commitment to the success of the operation?

  1. A) angel investors
  2. B) peer-to-peer lenders
  3. C) venture capitalists
  4. D) banks
  5. E) friends and family

Answer: D

Explanation: D) Lenders often insist that entrepreneurs put up as much as 30 percent of startup costs as proof of commitment to a new business.

Page Ref: 180

Difficulty: Moderate

Objective: 6.4 What are various ways of financing my new business?

Classification: Concept

61) Which source of financing for new businesses does NOT involve money?

  1. A) bank loans
  2. B) bartering
  3. C) angel investing
  4. D) peer-to-peer lending
  5. E) venture capital

Answer: B

Explanation: B) Bartering is the trading of goods or services without exchanging money. This can help new business owners obtain resources from suppliers without draining cash.

Page Ref: 181

Difficulty: Moderate

Objective: 6.4 What are various ways of financing my new business?

Classification: Concept

62) Which of the following tools for supporting business is NOT associated with the government?

  1. A) credit union
  2. B) enterprise zone
  3. C) the Small Business Administration
  4. D) incubator
  5. E) the Small Business Investment Company

Answer: A

Explanation: A) Enterprise zones and incubators often have some government backing. The SBA is a federal agency and the SBIC is a public/private partnership. Credit unions have no government link, other than being subject to standard financial institution regulations.

Page Ref: 181

Difficulty: Moderate

Objective: 6.4 What are various ways of financing my new business?

Classification: Concept

63) Sarita has been having difficulty getting a bank loan to help finance startup costs for her new food truck business. Which of the following can help a bank feel more confident in lending money to support Sarita’s new enterprise?

  1. A) Sarita’s business model
  2. B) Small Business Administration support
  3. C) urban enterprise zone participation
  4. D) references from family
  5. E) angel investor support

Answer: B

Explanation: B) The Small Business Administration guarantees many small business loans to banks.

Page Ref: 181

Learning Outcome: Explain the entrepreneurship process

Difficulty: Moderate

Objective: 6.4 What are various ways of financing my new business?

Classification: Application

64) Which of the following is NOT associated with financial institutions?

  1. A) credit unions
  2. B) the Small Business Administration
  3. C) banks
  4. D) peer-to-peer lending networks
  5. E) savings and loans

Answer: D

Explanation: D) Peer-to-peer lending groups are associated with private organizations or Web-based communities and offer microloans to support new businesses.

Page Ref: 182

Difficulty: Moderate

Objective: 6.4 What are various ways of financing my new business?

Classification: Concept

65) Which of the following is NOT a common method for financing startup businesses?

  1. A) venture capital
  2. B) credit cards
  3. C) angel investing
  4. D) going public
  5. E) bartering

Answer: D

Explanation: D) A public stock offering is not a method to finance a startup, but it might be done for a small company that has attained a substantial measure of success.

Page Ref: 182

Difficulty: Moderate

Objective: 6.4 What are various ways of financing my new business?

Classification: Concept

66) Which of the following would NOT commonly secure funding through a venture capitalist firm?

  1. A) medical device company
  2. B) biotech company
  3. C) green energy company
  4. D) landscaping company
  5. E) software company

Answer: D

Explanation: D) Venture capitalists provide little financing for small companies, and tend to focus on software, medical devices, biotech, green technology, and Internet-specific enterprises.

Page Ref: 182

Difficulty: Moderate

Objective: 6.4 What are various ways of financing my new business?

Classification: Application

67) Josh needs financial support to get his new custom kite business off the ground. Which of the following options does NOT involve a direct, up-front commitment of resources from Josh?

  1. A) credit cards
  2. B) second mortgage
  3. C) second job
  4. D) peer-to-peer lending
  5. E) personal savings

Answer: D

Explanation: D) Peer-to-peer lending is offered by outside groups and organizations. Josh will eventually need to pay back a peer loan, but it does not require an up-front commitment of his money, credit, or time.

Page Ref: 182

Difficulty: Moderate

Objective: 6.4 What are various ways of financing my new business?

Classification: Application

68) In which type of business support do tax credits play a significant role?

  1. A) public stock offerings
  2. B) Small Business Administration loan guarantees
  3. C) enterprise zones
  4. D) supplier bartering arrangements
  5. E) peer-to-peer lending networks

Answer: C

Explanation: C) In enterprise zones, governments try to encourage businesses to locate in a designated area through lower taxes and other incentives.

Page Ref: 182

Learning Outcome: Explain the entrepreneurship process

Difficulty: Moderate

Objective: 6.4 What are various ways of financing my new business?

Classification: Concept

69) Which source of financing has the greatest influence to force a business owner to sell the enterprise to a competitor and collect a high financial return?

  1. A) banks
  2. B) family
  3. C) angel investors
  4. D) peer-to-peer lenders
  5. E) venture capitalists

Answer: E

Explanation: E) Because venture capitalists get an ownership stake in the businesses they support—and sometimes a majority stake—they have a strong influence on the decision to sell a successful business to profit from their investment.

Page Ref: 182

Difficulty: Moderate

Objective: 6.4 What are various ways of financing my new business?

Classification: Synthesis

70) Yuri utilized every possible financing option to get his new business started. Now, after two years and fantastic growth, he is preparing to sell the business at a substantial profit. Which of the following will NOT realize additional profit from Yuri’s sale of the business?

  1. A) family investors
  2. B) bank lender
  3. C) angel investors
  4. D) venture capitalists
  5. E) friendly investors

Answer: B

Explanation: B) Angel investors and venture capitalists expect to reap profits from the sale of a business. Family and friends should be offered some type of additional compensation for supporting Yuri in his startup days. A bank will only have its loan repaid and will earn the amount of interest reflected in the loan terms.

Page Ref: 182

Difficulty: Difficult

Objective: 6.4 What are various ways of financing my new business?

Classification: Application

71) Palash wants to draw upon a variety of resources to help support his new digital printing business. He would like to take advantage of significant financial support from the government. Which of the following is the best option to help Palash meet this goal?

  1. A) working with the Small Business Administration
  2. B) getting space in an incubator
  3. C) going public and selling stock
  4. D) locating in an enterprise zone
  5. E) securing venture capital

Answer: D

Explanation: D) Governments offer substantial tax credits and other forms of support to businesses that locate in enterprise zones.

Page Ref: 182

Learning Outcome: Explain the entrepreneurship process

Difficulty: Moderate

Objective: 6.4 What are various ways of financing my new business?

Classification: Application

72) David embraced the bedouin work style when he began his consulting business, but now he feels ready for a little permanence and resource support. Which of the following will best meet his needs?

  1. A) angel investor
  2. B) incubator
  3. C) venture capitalist
  4. D) enterprise zone
  5. E) public stock offering

Answer: B

Explanation: B) As a bedouin in the modern business world, David works via laptop and mobile phone in various locations, such as cafes. The next level of permanence and support is an incubator, which provides low-cost office space and basic support services.

Page Ref: 182

Difficulty: Moderate

Objective: 6.4 What are various ways of financing my new business?

Classification: Synthesis

73) Which of the following reasons for the failure of a small business is NOT grounded in financial challenges?

  1. A) problems with cash flow
  2. B) unfavorable economies of scale
  3. C) highly competitive market
  4. D) lack of credit
  5. E) initial underfunding

Answer: C

Explanation: C) A very competitive market may cut into profits for a small business but is not directly connected to the finances needed for a business to operate successfully.

Page Ref: 184

Difficulty: Moderate

Objective: 6.5 How can I avoid failure and keep my business healthy?

Classification: Concept

74) One of the cardinal rules of business is that the customer is always right. Which of the following business activities is NOT associated with this philosophy?

  1. A) develop familiarity with clientele
  2. B) implement economies of scale
  3. C) adapt to the market’s changing needs
  4. D) set fair prices
  5. E) sell quality products

Answer: B

Explanation: B) Economies of scale are primarily related to the financial aspects of business operations.

Page Ref: 184

Learning Outcome: Explain the entrepreneurship process

Difficulty: Moderate

Objective: 6.5 How can I avoid failure and keep my business healthy?

Classification: Concept

75) A small business can fail for many reasons. Which of the following reasons involves financial penalties?

  1. A) inadequate management skills
  2. B) lack of a business plan
  3. C) failure to file appropriate paperwork
  4. D) inability to weather aggressive competition
  5. E) difficulty retaining good employees

Answer: C

Explanation: C) Small business owners who do not have expert accounting and/or legal assistance often have difficulty with the extensive government paperwork requirements for operating a business. Missing deadlines can result in penalties and fines.

Page Ref: 184

Difficulty: Moderate

Objective: 6.5 How can I avoid failure and keep my business healthy?

Classification: Concept

76) Last year Sara made a lot of money selling organic sandwiches from her food truck parked in front of the local college. After the winter snows melted, she was surprised to see seven other food trucks parked nearby, and shorter lines and lower profits for her business. What best describes the challenge she faces?

  1. A) losing touch with customers
  2. B) aggressive competition
  3. C) unfavorable economies of scale
  4. D) inadequate management skills
  5. E) lack of credit

Answer: B

Explanation: B) Competitors recognized the success of Sara’s location and moved into the market, cutting into her sales and profits.

Page Ref: 184

Learning Outcome: Explain the entrepreneurship process

Difficulty: Moderate

Objective: 6.5 How can I avoid failure and keep my business healthy?

Classification: Application

77) Teena owns a moderately successful Cuban restaurant, but finances are always tight. Her chef is extremely talented and a key factor in the restaurant’s success, but he has been considering offers from other restaurants. What is Teena’s best strategy to retain this important asset?

  1. A) offer the chef additional vacation time and a year-end bonus
  2. B) give the chef a new management-oriented title
  3. C) give the chef a raise and buy lower quality ingredients
  4. D) offer the chef an ownership stake in the restaurant
  5. E) let the chef leave and hire one who will be more loyal

Answer: D

Explanation: D) Retaining good employees can be a problem for small businesses that have limited finances and little room for professional growth. Offering partial ownership to valued employees gives them additional potential earnings and can increase their commitment.

Page Ref: 184

Learning Outcome: Explain the entrepreneurship process

Difficulty: Moderate

Objective: 6.5 How can I avoid failure and keep my business healthy?

Classification: Application

78) Which of the following is NOT a common reason that small businesses fail?

  1. A) lack of financial support
  2. B) government paperwork requirements
  3. C) difficulty retaining good employees
  4. D) lack of access to technology
  5. E) aggressive competition

Answer: D

Explanation: D) Technology capabilities are not a common factor behind the failure of small businesses.

Page Ref: 183

Learning Outcome: Explain the entrepreneurship process

Difficulty: Moderate

Objective: 6.5 How can I avoid failure and keep my business healthy?

Classification: Concept

79) What distinguishes a micropreneur from an entrepreneur?

Answer: Generally, a micropreneur wants to stay small and make a living, while an entrepreneur may begin as a small business but hope to grow into a larger enterprise.

Explanation: Entrepreneurs tend to focus on growth, whereas micropreneurs often are motivated by having more control and a balanced lifestyle.

Page Ref: 163

Learning Outcome: Describe the role of small businesses in the economy

Difficulty: Moderate

Objective: 6.1 Do I have what it takes to be a small-business entrepreneur?

Classification: Concept

80) WaterPod Transport is the only company in the world to manufacture and sell personal submarines. The company is owned by a wealthy entrepreneur and has 50 employees and annual sales revenue of $3 million. Would the U.S. Small Business Administration consider WaterPod to be a small business?

Answer: No, the SBA would not consider WaterPod Transport to be a small business because the company is dominant in its field of operation. No other company makes or sells personal underwater transportation.

Explanation: A company must not be dominant in its field of operation in order for it to be defined as a small business by the SBA.

Page Ref: 163

Difficulty: Moderate

Objective: 6.1 Do I have what it takes to be a small-business entrepreneur?

Classification: Application

81) A group of recent college grads is working to come up with “the next big thing.” Considering the definition of an entrepreneurial team, what should these people bring to the table to help ensure the success of their effort?

Answer: The team members should contribute different types of expertise to the effort.

Explanation: An entrepreneurial team is a group of people with different types of expertise coming together to work on a new project. If all team members have the same expertise, key aspects of the project may suffer.

Page Ref: 166

Difficulty: Moderate

Objective: 6.1 Do I have what it takes to be a small-business entrepreneur?

Classification: Application

82) Why does entrepreneurship tend to increase during economic recessions?

Answer: People who can’t find jobs in a bad economy can be under pressure to earn income and many become necessity entrepreneurs.

Explanation: In 2008, during the Great Recession, entrepreneurial activity increased 6 percent. Needing income in a recession, people become more entrepreneurial.

Page Ref: 167

Difficulty: Moderate

Objective: 6.1 Do I have what it takes to be a small-business entrepreneur?

Classification: Concept

83) Greg recently graduated from college with a degree in marketing. Where is his best opportunity to land his first professional job, and why?

Answer: Greg’s best chance for getting his first job is with a small company. Small businesses provide the first job for most entrants to the labor force.

Explanation: Small firms provide the first job for most entrants to the labor force, and generated 64 percent of all net new jobs annually for 15 years up to 2008.

Page Ref: 168

Difficulty: Moderate

Objective: 6.2 How important is small business, and what newer types might interest me?

Classification: Application

84) In the modern U.S. economy, what are bedouins?

Answer: Bedouins are people who use their laptops and mobile phones to operate small businesses out of cafes, coffeehouses, and other nonfixed locations.

Explanation: New computer and telecommunications technology enables “bedouin” businesspeople to be highly mobile; many have no physical office.

Page Ref: 169

Difficulty: Moderate

Objective: 6.2 How important is small business, and what newer types might interest me?

Classification: Concept

85) Google ads and banner ads are two aspects of the economic infrastructure that enables websites to easily make money. What is the third, and how does it operate?

Answer: Affiliated marketing is the third method for a website to easily make money. Under this arrangement, other Web-based businesses pay the initial website to refer customers.

Explanation: Affiliated marketing enables a website to easily make money. Under this structure, other Web-based businesses pay the initial website to refer customers, usually through links on the website.

Page Ref: 171

Difficulty: Moderate

Objective: 6.2 How important is small business, and what newer types might interest me?

Classification: Concept

86) What is the potential financial benefit to buying an existing business?

Answer: An existing, well-run enterprise should generate profits quickly.

Explanation: Buying an existing enterprise that is already successful should generate profits quickly for the buyer. By comparison, starting a new business may not generate profits for several years.

Page Ref: 173

Difficulty: Moderate

Objective: 6.2 How important is small business, and what newer types might interest me?

Classification: Concept

87) What is considered to be a key element in making a small, Web-based business successful?

Answer: Exploiting a niche by offering a unique product or service not easily available to potential customers can lay solid groundwork for a Web-based business idea.

Explanation: Exploiting a niche plays a key role in the success of any small business, but is particularly true for Web-based businesses.

Page Ref: 175

Learning Outcome: Explain the entrepreneurship process

Difficulty: Moderate

Objective: 6.3 If I wanted to start a company, how should I go about it?

Classification: Concept

88) Darla thinks she has found her small-business niche: nail polish for miniature dogs. She loves mixing her own polish colors and adores teacup doggies, and her friends say she gives great manicures. What else should Darla consider to create a successful business?

Answer: Darla should be sure there is a need in the marketplace for her doggie nail polish idea.

Explanation: A good business idea should involve something you like doing and are good at, should exploit a niche, and should satisfy some sort of need in the marketplace.

Page Ref: 175

Learning Outcome: Explain the entrepreneurship process

Difficulty: Moderate

Objective: 6.3 If I wanted to start a company, how should I go about it?

Classification: Application

89) Kay is excited about her idea to open an organic cafe, but she wants to know all about the realities of operating a restaurant. She waited tables in college and has met with a retired food service executive who arranged a mentor for her. What other resource can Kay use before writing her business plan?

Answer: Kay can get data, training, and additional contacts through a restaurant trade association.

Explanation: Kay has done all the right things: working in restaurants, getting a mentor, meeting with people who have industry management experience. A trade association is a group promoting common interests in an industry. This can help fill any research gaps Kay might have.

Page Ref: 176

Learning Outcome: Explain the entrepreneurship process

Difficulty: Moderate

Objective: 6.3 If I wanted to start a company, how should I go about it?

Classification: Application

90) Monique is writing a list of questions to consider as she drafts her business plan. She is entering a very competitive market served by many existing companies. What is the most important question she needs to answer to help secure funding and get her business started?

Answer: The most important question for Monique to answer is “How is my business idea different from existing competitors?”

Explanation: Monique is attempting to enter a saturated, competitive market. Many questions are important to consider in preparing a business plan, but given the context of the situation, Monique must consider how her business will be distinct from competitors.

Page Ref: 178

Learning Outcome: Explain the entrepreneurship process

Difficulty: Moderate

Objective: 6.3 If I wanted to start a company, how should I go about it?

Classification: Application

91) What is the difference between an angel investor and a venture capitalist?

Answer: An angel investor is a wealthy individual who wants to make an investment, whereas a venture capitalist is typically a company who wants a stake in the company in return for part ownership.

Explanation: Angel investors and venture capitalist can both make an investment in a new business, but venture capitalists are typically looking for part ownership in the company.

Page Ref: 182

Learning Outcome: Explain the entrepreneurship process

Difficulty: Moderate

Objective: 6.4 What are various ways of financing my new business?

Classification: Concept

92) Why do governments encourage new businesses to locate in urban enterprise zones?

Answer: Urban enterprise zones are located in economically distressed areas in industrial or commercial parts of cities. By drawing new business into enterprise zones, governments hope to revitalize the areas.

Explanation: Governments offer a variety of tax credits and other incentives to encourage businesses to locate in enterprise zones. The governments hope to benefit from tax revenues and jobs that the businesses will create.

Page Ref: 182

Difficulty: Moderate

Objective: 6.4 What are various ways of financing my new business?

Classification: Concept

93) Jorge still needs a large amount of financing for his new business. He has already tapped into his credit cards and personal contacts and has secured a bank loan. He is intent on retaining full ownership of the company. What financing option should he pursue?

Answer: To obtain substantial levels of funding without giving up an ownership stake in his new company, Jorge should seek out angel investors.

Explanation: Angel investors are individuals who have a lot of money and are looking to make more by investing in companies with high growth potential.

Page Ref: 181

Difficulty: Moderate

Objective: 6.4 What are various ways of financing my new business?

Classification: Application

94) Kelly’s new landscape architecture firm is located in a large downtown loft space shared with other small businesses. All the tenants are served by a central, low-cost administrative support crew and have access to conference rooms. Where is Kelly’s firm operating?

Answer: Kelly’s firm is operating in an incubator, which provides low-cost office space and basic services to help new business enterprises get established.

Explanation: An incubator is a facility that offers small businesses low-cost offices with basic services such as legal and secretarial. Incubators are often sponsored by government economic development departments.

Page Ref: 182

Difficulty: Moderate

Objective: 6.4 What are various ways of financing my new business?

Classification: Application

95) Like many new business owners, Nattaya is having difficulty with cash flow in her Thai restaurant. She just learned she needs a new stove hood to comply with health codes, but the cost is prohibitive. What option can she pursue to get a new hood without spending cash?

Answer: Nattaya can offer to barter with a ventilation company. In return for installing a new stove hood, she could provide free lunches to the company’s employees for a specified period or offer to cater its holiday party.

Explanation: Bartering is trading goods or services without exchanging money.

Page Ref: 181

Difficulty: Moderate

Objective: 6.4 What are various ways of financing my new business?

Classification: Application

96) Giancarlo has run a successful Italian trattoria for nearly 30 years. When asked the secret to his success, he always attributes it to his loyal customers. What basic philosophy does Giancarlo follow to keep his customers happy and coming back for more good food?

Answer: To maintain a satisfied customer base, Giancarlo sells a quality product at a fair price.

Explanation: The best way to keep customers happy is to sell a quality product or service at a fair price.

Page Ref: 184

Learning Outcome: Explain the entrepreneurship process

Difficulty: Moderate

Objective: 6.4 What are various ways of financing my new business?

Classification: Application

97) Why does paperwork often present a problem to small businesses?

Answer: They often have a lack of specialists to deal with it.

Explanation: Small-business owners can be easily overwhelmed with filing the necessary government paperwork for their business as well as attending to the other needs necessary to run a small business.

Page Ref: 184

Learning Outcome: Explain the entrepreneurship process

Difficulty: Moderate

Objective: 6.5 How can I avoid failure and keep my business healthy?

Classification: Concept

98) There are many reasons why small businesses fail. What reason is a direct reflection of the business owner’s capabilities?

Answer: An entrepreneur’s inadequate management skills are a key reason for the failure of small businesses.

Explanation: Running a small business requires an entrepreneur to wear many hats, from sales and marketing to strategic planning and cost control. Lacking capabilities in a crucial area can lead to failure of the business.

Page Ref: 183

Learning Outcome: Explain the entrepreneurship process

Difficulty: Moderate

Objective: 6.5 How can I avoid failure and keep my business healthy?

Classification: Concept

99) What are three important actions an entrepreneur should take to keep a small business healthy?

Answer: An entrepreneur should keep accurate financial records, stay in touch with customers, and effectively manage employees.

Explanation: To run a successful small business, an entrepreneur should keep accurate financial records, stay in touch with customers, and effectively manage employees.

Page Ref: 184

Learning Outcome: Explain the entrepreneurship process

Difficulty: Moderate

Objective: 6.5 How can I avoid failure and keep my business healthy?

Classification: Concept

100) Many small businesses fail because they are selling products or services that are new and untried. This sounds like the definition of innovation. Yet, why does this often lead to failure of an enterprise?

Answer: By selling products and services that are new and untried, the business is assuming a greater-than-average level of risk.

Explanation: All businesses operate in a climate of risk. Small businesses attempting to sell new products or services face additional risks and often lack adequate resources to weather challenges that a larger business selling new products and services may be able to manage.

Page Ref: 184

Difficulty: Moderate

Objective: 6.5 How can I avoid failure and keep my business healthy?

Classification: Concept

An entrepreneur, Christina Rossini, is planning to open a small chain of 4 to 6 pizza stores under the brand name Redenda Pizza. The name comes from combining the first names of the entrepreneur’s grandparents. Christina wants to have a competitive advantage by offering innovative and distinct specialty pizzas and by offering the freshest ingredients. However, she is concerned that without proper planning, her venture will not be successful.

101) Answers to which of the following questions are LEAST relevant to determining whether Christina has located a good opportunity in starting Redenda Pizza?

  1. A) Are there other pizza restaurants in the immediate vicinity of the planned locations?
  2. B) Is the business of other pizza restaurants in the city growing?
  3. C) Do the planned locations of the restaurant have competitive rents?
  4. D) Will Redenda Pizza offer unique ingredients in its food offerings?
  5. E) Would Christina bring unique talents and ideas to a restaurant business?

Answer: C

Explanation: C) The rents of the planned locations are not of direct relevance to how good the opportunity is because the success of the restaurant is independent of how good the rents are. Even if the rents are competitive, that would not be sufficient for the success of the restaurant. On the other hand, the questions of whether there is sufficient demand for pizza restaurants and not too much supply of them are directly relevant to the question of whether there is a good opportunity for a pizza restaurant, as is the question of whether Christina will bring unique things to a pizza restaurant that would enhance its chances of success.

Page Ref: 163

Difficulty: Moderate

Objective: 6.1 Do I have what it takes to be a small-business entrepreneur?

Classification: Critical Thinking

AACSB: Reflective Thinking Skills

102) Which of the following, if true, strengthens the case that Christina has a good opportunity in opening a new pizza restaurant?

  1. A) There are many existing pizza restaurants in Christina’s city.
  2. B) The pizza business in Christina’s city has remained largely unchanged for many years.
  3. C) Restaurant consumers in Christina’s city mostly frequent Mexican restaurants.
  4. D) Pizza restaurants require several experienced employees to be successful.
  5. E) Pizza restaurants can be found in almost any town in the United States.

Answer: B

Explanation: B) If the pizza business has remained largely unchanged for a period of time, this represents an opportunity for Christina to bring something different to the local industry and thereby attract customers because of the newness involved. Choice A indicates that consumer needs are already met by other establishments, and Choice C implies that demand for pizza restaurants is low in Christina’s city, indicating little opportunity. Choices D and E do not strengthen the case of a pizza restaurant being a good opportunity for Christina.

Page Ref: 163

Difficulty: Moderate

Objective: 6.1 Do I have what it takes to be a small-business entrepreneur?

Classification: Critical Thinking

AACSB: Reflective Thinking Skills

103) Which of the following, if true, does NOT strengthen the case that Christina has personal characteristics necessary for her new business to succeed?

  1. A) She is willing to take risks.
  2. B) She is motivated.
  3. C) She works well with others.
  4. D) She is a “big picture” thinker.
  5. E) She follows directions carefully.

Answer: E

Explanation: E) Carefully following directions may be a useful general characteristic for a worker, but it is of no particular help to an entrepreneur. Rather, good leadership characteristics contribute to being a successful entrepreneur and being able to work well with a variety of people involved in the business. A good entrepreneur must be willing to take risks or there will be no chance to start a business, which is inherently risky. It takes motivation to see a new business through to success, and “big-picture” thinking helps to keep all the factors in mind that are necessary for a business’s success.

Page Ref: 163

Difficulty: Moderate

Objective: 6.1 Do I have what it takes to be a small-business entrepreneur?

Classification: Critical Thinking

AACSB: Reflective Thinking Skills

104) Which of the following best explains why flexibility is an important quality that Christina needs for success?

  1. A) It takes a certain amount of money to start a business.
  2. B) It often helps to have a business partner in a new business.
  3. C) Starting a new business requires a great deal of planning.
  4. D) Starting and running a new business involve many unpredictable factors.
  5. E) An entrepreneur must be extremely competent in business skills.

Answer: D

Explanation: D) Starting a new business requires planning but also the ability to adjust to the inevitable eventualities in which things do not go according to plan and to improvise so that goals can still be reached. Flexibility does not directly relate to acquiring startup money or to having a business partner or business skills.

Page Ref: 163

Difficulty: Difficult

Objective: 6.1 Do I have what it takes to be a small-business entrepreneur?

Classification: Critical Thinking

AACSB: Reflective Thinking Skills

Lola’s Soft Drinks is a startup soda pop company that produces kitschy flavors like pineapple and jalapeño. The small company’s competitive advantage rests in its owners’ narrative: A few college friends liked to get together on weekends and make homemade soft drinks. During their senior year, the friends learned about bottling and sold cases of soda to fellow students. Sales exceeded expectations, and Lola’s Soft Drinks was born. All of the original owners still work hands-on at the soda factory.

Now that all of the owners of Lola’s Soft Drinks have started families, some of them want to hire more staff to reduce their personal workload and to take on roles that focus on being executives and overseers rather than the hands-on roles they fill right now. However, one of the owners, Huan, thinks this is a bad idea. The soda company is based on the image of young entrepreneurs making their own product and personally selling it to the customers. According to Huan, if they let a staff of workers take over the operation of their business, Lola’s will be no different from the many other soft drink companies in their industry and will lose its unique niche.

105) Which of the following, if true, weakens Huan’s argument?

  1. A) No other companies produce the same flavors as Lola’s Soft Drinks.
  2. B) Sales of Lola’s Soft Drinks have been steadily increasing for the last three years.
  3. C) The company owners work well together and still get along nicely.
  4. D) The bottling facility currently used by the company is not large enough to increase production.
  5. E) The owners hope to pass their successful business on to their own children one day.

Answer: A

Explanation: A) Huan believes that changing staff will turn buyers off and encourage them to buy soft drinks from other companies. If Lola’s Soft Drinks is the only company that produces these flavors of sodas, however, its competitive advantage over other companies will still exist even if the staff changes are made. Choice B suggests that sales have steadily increased under the owners’ current operation, which suggests that major changes should be approached with caution. Choice C explains that the owners all work well together, which supports Huan’s argument. Choices D and E do not directly address the issue of whether the owners should hire new staff.

Page Ref: 164

Learning Outcome: Explain the entrepreneurship process

Difficulty: Moderate

Objective: 6.1 Do I have what it takes to be a small-business entrepreneur?

Classification: Critical Thinking

AACSB: Reflective Thinking Skills

106) All of the following questions are relevant to the owners’ decision about hiring new staff EXCEPT which one?

  1. A) Do the owners want to stop participating in the hands-on production of their product?
  2. B) How much will employing new staff members cost?
  3. C) Will new employees put the same amount of care into making the soda as the owners do?
  4. D) Will Lola’s be introducing new flavors in the upcoming year?
  5. E) Do customers buy the soda because they think it is made personally by the owners?

Answer: D

Explanation: D) Lola’s Soft Drinks can stick with the same flavors or introduce new flavors no matter who produces the soda, so Choice D is not relevant. Choice A is a relevant question because if the owners no longer want to do the hands-on work of producing the soda, they should hire other people to do it. Choice B is relevant because employing a staff may have significant effects on revenues and profits. Choice C is relevant because having a staff create the soda rather than the company owners might affect the products’ quality as well as customers’ perception of the product. Choice E is an important question to ask because if customers do not care who actually produces the soda, the owners can hire staff without affecting sales.

Page Ref: 164

Learning Outcome: Explain the entrepreneurship process

Difficulty: Difficult

Objective: 6.1 Do I have what it takes to be a small-business entrepreneur?

Classification: Critical Thinking

AACSB: Reflective Thinking Skills

107) Which of the following is the point at issue between Huan and his co-owners?

  1. A) whether the soda company will ultimately be profitable enough to sell
  2. B) if the soda company should be sold right away
  3. C) whether the soda company can gain new customers
  4. D) if the owners should hire staff
  5. E) whether the owners have a viable business

Answer: D

Explanation: D) Huan and his co-owners disagree over whether they should hire staff to maintain the daily operations of their business. Choices A and B relate to selling the company, and we don’t know how any owners feel about that. Huan and the other owners presumably agree on Choices C and E, that the company can gain new customers and that it’s currently a viable business.

Page Ref: 164

Learning Outcome: Explain the entrepreneurship process

Difficulty: Easy

Objective: 6.1 Do I have what it takes to be a small-business entrepreneur?

Classification: Critical Thinking

AACSB: Reflective Thinking Skills

Short and Shearer are two friends who want to begin an entrepreneurship as business partners. They need to decide whether to start up a new business or to buy an existing business. To do this, Short and Shearer need to weigh the pros and cons of each strategy in order to see which is the best strategy for them.

108) Which of the following, if true, strengthens the case for Short and Shearer buying an existing business?

  1. A) Short and Shearer have a lot of ideas as to the kind of business they want to own.
  2. B) An existing business would have an existing customer base that Short and Shearer would inherit.
  3. C) An existing business for sale might have difficulties that Short and Shearer would inherit.
  4. D) Short and Shearer greatly value independence.
  5. E) Existing small businesses usually carrying debts.

Answer: B

Explanation: B) Having an existing customer base would make doing business easier because they could count on a relatively reliable source of revenue right away. If an existing small business has debts or other difficulties that Short and Shearer would inherit, this weakens the case for buying it. If Short and Shearer value independence, this means they should be entrepreneurs, but it does not strengthen the case for their buying an existing business over starting their own. If they have a lot of business ideas, this does not strengthen the case for buying an existing business, although it would strengthen the case for them starting a new business of their own.

Page Ref: 173

Difficulty: Moderate

Objective: 6.2 How important is small business, and what newer types might interest me?

Classification: Critical Thinking

AACSB: Reflective Thinking Skills

109) Which of the following, if true, weakens the case for Short and Shearer buying an existing business?

  1. A) Short and Shearer could more easily obtain financing for the purchase.
  2. B) Buying an existing business involves fewer legal hurdles than starting a new one.
  3. C) Franchises have more potential for success than single-facility businesses.
  4. D) Short and Shearer have limited funds to start with.
  5. E) Existing businesses cost less to purchase than new ones.

Answer: D

Explanation: D) If Short and Shearer have limited funds, they are probably not in a good position to pay the full purchase price for an existing business, which tends to have a lot of existing value. If, however, it costs less, they could easily obtain financing, or it involves fewer legal hurdles, these strengthen the case for buying an existing business. It is not given whether Short and Shearer are looking at buying a franchise or a single-facility business, so if a franchise is preferable, this does not weaken the case for their buying an existing business.

Page Ref: 173

Difficulty: Moderate

Objective: 6.2 How important is small business, and what newer types might interest me?

Classification: Critical Thinking

AACSB: Reflective Thinking Skills

110) Which of the following, if true, strengthens the case for Short and Shearer’s starting a new business of their own?

  1. A) Starting a new business puts a lot of demands on their time.
  2. B) Starting a new business involves more risk than purchasing an existing one.
  3. C) Starting a new business gives them a lot of freedom and independence.
  4. D) Starting a new business means hiring many new employees right away.
  5. E) Starting a new business involves a lot of stress for the proprietors.

Answer: C

Explanation: C) If Short and Shearer are entrepreneurs, this makes it highly likely that they value freedom and independence, which strengthens the case for starting a new business of their own. Risk, stress, demands on their time, and having to hire several new employees are all weaknesses to the case being made.

Page Ref: 173

Difficulty: Moderate

Objective: 6.2 How important is small business, and what newer types might interest me?

Classification: Critical Thinking

AACSB: Reflective Thinking Skills

111) Which of the following, if true, weakens the case for Short and Shearer’s starting a new business?

  1. A) Starting a new business allows entrepreneurs to determine their own working hours.
  2. B) Starting a new business allows entrepreneurs to establish their own clientele.
  3. C) Starting a new business allows entrepreneurs to use their own ideas.
  4. D) Starting a new business gives a potential for a feeling of great accomplishment.
  5. E) Starting a new business involves using an untested business model.

Answer: E

Explanation: E) An untested business model involves risk to the entrepreneur since it increases the chances that it might fail. Determining one’s own working hours and clientele, using one’s own ideas, and having the potential for gaining a sense of accomplishment strengthen the case for starting a new business.

Page Ref: 173

Difficulty: Moderate

Objective: 6.2 How important is small business, and what newer types might interest me?

Classification: Critical Thinking

AACSB: Reflective Thinking Skills

112) In order to help ensure success for their new business, which of the following questions is LEAST helpful for Short and Shearer to periodically ask themselves?

  1. A) How well does our firm’s operation align with the business plan?
  2. B) How many employees do we have?
  3. C) How are we handling our finances?
  4. D) How are our employees performing?
  5. E) How much debt can we afford to carry?

Answer: B

Explanation: B) The number of employees in itself is not an indicator of a business’s health. The business could be doing very well with few employees or floundering with many employees (or vice versa). On the contrary, keeping to the business plan, handling finances wisely, ensuring that employees are performing well, and making sure that the amount of debt the business is carrying is appropriate are all very helpful, if not essential, questions that small-business owners should ask themselves periodically.

Page Ref: 173

Difficulty: Difficult

Objective: 6.2 How important is small business, and what newer types might interest me?

Classification: Critical Thinking

AACSB: Reflective Thinking Skills

Many companies start as nichers to get a foothold against larger competitors and then grow into broader competitors. Enterprise Rent-A-Car began by building a network of neighborhood offices rather than competing with Hertz and Avis in airport locations. Enterprise is now the nation’s largest car rental company.

113) Which of the following is best supported by the statements above?

  1. A) The best path to long-term growth is to avoid confrontation with larger competitors for as long as possible.
  2. B) It is generally better to confront one established competitor at a time rather than many at once.
  3. C) Even when competing in a niche, a company should act in every respect as if it is targeting every market segment.
  4. D) The car rental industry is the best example for niche companies to consider when evaluating how to grow into prominence.
  5. E) A company’s initial goals do not place limits on its long-term success.

Answer: E

Explanation: E) The only safe inference here is Choice E. Enterprise started small and had modest aims, but it is now the largest car rental company. So the case of Enterprise is evidence for the notion that your initial goals don’t determine your ultimate fate. Choice A goes too far. It’s not even clear that Enterprise avoided confrontation as long as possible, and even if it did, Choice A’s claim is too strong. The case is about Enterprise and not what must be the case for all companies. Choice B: We don’t know if Enterprise took on competitors one at a time or all at once. Choice C goes against the passage. Enterprise started local. Choice D: This might be a good example, but there may be better ones out there.

Page Ref: 175

Difficulty: Difficult

Objective: 6.3 If I wanted to start a company, how should I go about it?

Classification: Critical Thinking

AACSB: Reflective Thinking Skills

114) In what way was Enterprise Rent-A-Car’s history as a niche company likely to have been a disadvantage as it sought to become an industry leader?

  1. A) It had less brand awareness than an entirely new company would have had.
  2. B) It lacked experience developing profitable customer relationships.
  3. C) It had to overcome its reputation as a “neighborhood” rental agency.
  4. D) It needed to end relationships with its existing customers.
  5. E) It needed to compete solely on price.

Answer: C

Explanation: C) Enterprise started out as a “neighborhood” company, and that might have helped at first, but a large part of the market probably wants a global company. So for them, Enterprise had to overcome the reputation that it spent years developing, as Choice C indicates. Choices A and B were less of a problem for Enterprise than they would be for new companies. Choices D and E are too extreme to be correct.

Page Ref: 175

Difficulty: Difficult

Objective: 6.3 If I wanted to start a company, how should I go about it?

Classification: Critical Thinking

AACSB: Reflective Thinking Skills

Aardvark is a small pest-control business that has 30 employees. The company started business within the past year and needs to make a decision on how best to make sure its finances are managed as the company continues to grow its customer base and its business.

115) Aardvark needs some short-term financing to meet a cash-flow gap while it makes immediate capital expenditures. Which of the following, if true, strengthens the case for the company using a credit card for this purpose?

  1. A) Family members of the company owners have a lot of liquid assets.
  2. B) The company is not sure when it will be able to repay the money.
  3. C) The company expects to be able to repay the money next month.
  4. D) The company needs to obtain a line of credit without interest.
  5. E) The company can provide collateral for a secured loan.

Answer: C

Explanation: C) Business credit cards can be a viable source of very short-term financing if the company expects to pay off the balance within a month, thus avoiding potentially high interest charges. If the money can’t be repaid within a month (Choice B), then family members (Choice A) who are the source of short-term loans are preferable to a credit card advance. The line of credit from a credit card is without interest for only a very limited time—usually a month or less, depending on the card (Choice D). Choice E: Having collateral strengthens the case that the company could obtain a bank loan, in which case it would not need to use a credit card.

Page Ref: 182

Learning Outcome: Explain the entrepreneurship process

Difficulty: Difficult

Objective: 6.4 What are various ways of financing my new business?

Classification: Critical Thinking

AACSB: Reflective Thinking Skills

Village Inn is a locally owned family-style restaurant in a well-travelled area, but it has experienced steadily declining business. Surveys indicate dissatisfaction with the quality of the food compared to similar restaurants within easy driving distance. The restaurant’s owner considers hiring a head chef from one of his competitors but determines that the balance sheet of the restaurant would better support hiring a less experienced but highly talented applicant.

Given the restaurant’s likely clientele, he wants a chef who is skilled at preparing meals that appeal to customers who expect a satisfying but unadventurous bill of fare. Yet, at the same time, he hopes for someone who will find ways to make Village Inn’s food seem “special good” without introducing flavors or dishes that might challenge the palates of regular patrons and their friends. In addition, he wants a chef with some supervisory experience who will take charge of the increased number of part-time cooks and servers that the restaurant hired for peak times and seasons.

116) Given the owner’s priorities, which of the following, if true, most clearly justifies posting a job description on a local Internet job board as the most effective first step?

  1. A) The restaurant is located in an area served by a community college with a popular culinary arts program.
  2. B) Posting a notice on a web-based job board is less expensive than running print advertisements in the local papers.
  3. C) The restaurant’s owner is very active in web-surfing and would check the job board regularly for responses.
  4. D) Many young people in the area can now access online job boards by using smartphones.
  5. E) A web-based job description can be edited more quickly as the employer develops a better sense of whom he wants to recruit.

Answer: A

Explanation: A) A local job board will be effective only if it is read by applicants who have the interests, expertise, and experience that the employer expects. So Choice A is correct: if the college’s culinary program is thriving, then this should create a steady supply of chefs with training and some on-the-job experience. The other choices do not as clearly justify web over traditional print or word of mouth. Choices B and C relate to the employer’s convenience, but a cheap way of publicizing a job that is easy to check may not attract applications from qualified and motivated applicants. If facility with a smartphone were one of the skills desired, then Choice D might be a possibility, but technological facility is not directly linked with the abilities needed to function as a chef. Choice E misses the point: the employer should have a clear sense of the ideal applicant before posting the job description, not after.

Page Ref: 184

Difficulty: Difficult

Objective: 6.5 How can I avoid failure and keep my business healthy?

Classification: Critical Thinking

AACSB: Reflective Thinking Skills

117) The Village Inn owner narrows the field to three likely candidates for the head chef position and calls them in for a face-to-face interview. Which of the following topics, if raised by the employer in an interview, is the least helpful in identifying the best applicant?

  1. A) what they most dislike about preparing food in a family restaurant
  2. B) how they would go about preparing two or more complicated dishes simultaneously
  3. C) what kinds of group activities they were involved in during college
  4. D) what seasonings they would use in a meatloaf and gravy special
  5. E) how well they did in writing or public speaking courses

Answer: E

Explanation: E) In an entrepreneurial setting, it is best to focus quickly on factors that relate directly to the requirements of the job. So Choice E is correct: since the position does not require writing or public speaking, then the applicants’ responses to this topic are less helpful than ones more directly related to their suitability for the job. The other choices would elicit answers that relate to the factors that an entrepreneur should consider when hiring an employee. Choice A identifies important aspects of food preparation that the worker is (or is not) motivated to carry out, while Choice C allows the employer to gauge the applicant’s ability to work with other people. Choices B and D focus on specific work-related tasks and help determine if the applicant has the experience needed and the culinary approach needed to prepare common dishes that the restaurant’s patrons expect.

Page Ref: 184

Difficulty: Difficult

Objective: 6.5 How can I avoid failure and keep my business healthy?

Classification: Critical Thinking

AACSB: Reflective Thinking Skills

118) The Village Inn owner hires the candidate that impresses him as having the most potential for growing into the head chef position. He recognizes, however, that the new hire will need training to become familiar with the specific demands of the job. If the owner is a good manager, what would his first step be in devising a training program?

  1. A) take the new chef through the kitchen before opening time, showing the worker how to properly operate all the equipment safely
  2. B) develop a detailed job description, including the tasks a head chef must perform regularly and the skills needed to carry out each task
  3. C) present the worker with the restaurant’s employee manual and instruct him or her to read it carefully
  4. D) allow the new employee to observe the present kitchen staff in operation for a week before taking over the duties of the job
  5. E) enroll the new hire in a culinary arts webinar program to which the restaurant owner subscribes

Answer: B

Explanation: B) An effective training program relies on a clear understanding of the job duties and processes required to carry them out. So Choice B is correct: the owner should first devise a complete job description and then arrange for an appropriate format to make the new hire familiar with it. Each of the other choices might be appropriate in some cases given the restaurant’s resources and immediate needs and the chef’s previous experience, but Choice A does not adequately brief the employee on what the employer expects as results, while Choice C does not provide an opportunity for the worker to try out the new tasks. Choices D and E might help phase the new hire into the specific skills needed if the restaurant has the time and resources to provide them. All of these assume, however, that the employer has a clear set of expectations to communicate to the new chef, so the Choice B step would precede any of them.

Page Ref: 184

Difficulty: Difficult

Objective: 6.5 How can I avoid failure and keep my business healthy?

Classification: Critical Thinking

AACSB: Reflective Thinking Skills

119) The owner wishes to keep the training program informal and inexpensive and also allow the new chef to become familiar with the operations of the restaurant. Which of the following, if true, makes the owner’s job easier to carry out?

  1. A) The successful applicant graduated at the top of her culinary arts program at the community college.
  2. B) The new chef is being hired in June, which is usually a somewhat busy time for business at the restaurant.
  3. C) The new chef was born and raised in the community where the restaurant is based and so is familiar with local taste preferences.
  4. D) Several of the part-time cooks have worked at the restaurant for years and have, at times, assisted the former chefs at times of peak business.
  5. E) The restaurant’s cooking equipment was recently upgraded to make it safer to operate.

Answer: D

Explanation: D) While there are a number of informal training options that small businesses can devise, one of the strongest is creating an atmosphere in which employees share best practices in a free way. So Choice D is correct: if there are workers on the staff who are familiar with its operations, especially at stressful times, they are likely to be the most effective trainers (assuming they are not disgruntled at being passed over for the head chef job). The other choices all relate to facts that are convenient in some way, but do not necessarily make the job of training easier. Choices A and C, for instance, suggest that the applicant is intellectually sharp and has already absorbed many of the culinary preferences that the restaurant owner wants to stress. Neither talent, however, shows that the chef will be able to carry out the demands of the job under business pressures. Choice B gives the restaurant little space for inefficiency while the chef is picking up the trade, which does not bode well for the training process. Choice E obviously makes the workplace a safer place, but the chef (and the rest of the kitchen staff) will still need practice to learn to use the new equipment efficiently.

Page Ref: 184

Difficulty: Difficult

Objective: 6.5 How can I avoid failure and keep my business healthy?

Classification: Critical Thinking

AACSB: Reflective Thinking Skills

TestPrepWizard.com is a new test preparation company that offers online products and services helping students do their best on college and graduate school entrance exams. Awareness of the company is currently quite low. TestPrepWizard.com needs to decide which customers to serve. The company has decided to focus on high-end private tutoring services delivered over the Internet.

120) Which of the following, if true, best supports TestPrepWizard.com’s decision to focus on high-end private tutoring services?

  1. A) A significant proportion of the people interested in private tutoring to help them prepare for tests live far from tutors capable of providing high-end service.
  2. B) Most of the people who take standardized tests are unwilling to invest more than a modest amount in test preparation services.
  3. C) Organizations that provide admissions advice are more likely to be profitable than companies that offer financial aid advice.
  4. D) The number of students who prepare for college entrance exams is higher than the number of students who prepare for graduate entrance exams.
  5. E) In the current test preparation market, only a few companies have greater than a ten percent market share.

Answer: A

Explanation: A) One of the key decisions for any business is selecting customers to serve. TestPrepWizard.com has chosen to focus on high-end private tutoring. As an online company, it is able to reach anyone with an Internet connection. If Choice A is true, and lots of potential customers live too far from tutors, then those customers would be more likely to be interested in Internet-based tutoring. So Choice A supports the decision. Choice B weakens the argument by suggesting that most people wouldn’t pay for expensive tutoring. Choice C is about admissions advice and financial aid advice, which are outside of the scope of the argument here. Choice D suggests that college entrance exams could be a good market, but that doesn’t tell us anything about the market for private tutoring. Choice E says that there aren’t many big players in the market, but that doesn’t tell us what customers want.

Page Ref: 184

Learning Outcome: Explain the entrepreneurship process

Difficulty: Moderate

Objective: 6.5 How can I avoid failure and keep my business healthy?

Classification: Critical Thinking

AACSB: Reflective Thinking Skills

121) Which of the following, if true, most undermines TestPrepWizard.com’s decision to focus on high-end private tutoring services?

  1. A) Customers interested in test preparation services are increasingly likely to have used technology products in educational settings.
  2. B) Many students require extensive assistance in order to achieve their score goals on entrance exams.
  3. C) The market for high-end private tutoring services is especially competitive in urban areas.
  4. D) The graduate admission test with the largest number of test takers is also the test with the lowest rates of test preparation.
  5. E) Customers interested in high-end tutors will typically only hire a tutor recommended by a friend who has worked with that tutor.

Answer: E

Explanation: E) What most undermines the decision to go after the high-end tutoring market? If Choice E were true, and people insist on personal recommendations from someone who has used a tutor, then a new company would have a hard time breaking in. So Choice E weakens the argument. Choice A strengthens the argument by suggesting that technology won’t be a barrier. Choice B suggests that test prep is a good market but doesn’t tell us about high-end tutoring. Choice C says that the market is more competitive in urban areas, but as an online company, TestPrepWizard.com isn’t competing in just those areas. Choice D tells us about the market for prep for one test but doesn’t tell us about high-end tutoring.

Page Ref: 184

Learning Outcome: Explain the entrepreneurship process

Difficulty: Moderate

Objective: 6.5 How can I avoid failure and keep my business healthy?

Classification: Critical Thinking

AACSB: Reflective Thinking Skills

122) TestPrepWizard.com is considering whether to end its relationship with an especially demanding customer. Which of the following questions is LEAST relevant when making this decision?

  1. A) If the relationship ends, is the customer likely to send negative messages about TestPrepWizard.com to other potential customers?
  2. B) What contribution does this customer make to TestPrepWizard.com’s customer equity?
  3. C) What is the cost of the resources required to service this customer?
  4. D) Is it possible to identify especially demanding customers before any customer relationship exists?
  5. E) If the people who are serving the customer were reassigned to other projects, how would their reassignment affect the value of those projects?

Answer: D

Explanation: D) How do you decide whether you should “fire” your customer? The cost of the resources devoted to the customer (and the opportunity cost of those resources) is important, as is the amount of revenue the customer provides. Another important question is whether the unhappy customer will try to ruin your business. The irrelevant issue here is whether it was possible to identify the problem customer earlier on. That might be important for a future case, but knowing how to spot problem customers doesn’t help you decide what to do with the problem customers you already have.

Page Ref: 184

Learning Outcome: Explain the entrepreneurship process

Difficulty: Difficult

Objective: 6.5 How can I avoid failure and keep my business healthy?

Classification: Critical Thinking

AACSB: Reflective Thinking Skills

123) TestPrepWizard.com’s marketing department has produced a report that claims that even though virtually all customers believe that TestPrepWizard.com’s products and services have high value, a significant percentage of TestPrepWizard.com’s customers are not satisfied. A manager at TestPrepWizard.com takes issue with this claim, stating that this is impossible. Is the manager correct? Why or why not?

  1. A) Yes, because by definition when customer-perceived value is high, customers are satisfied.
  2. B) Yes, because successful companies deliver both high customer-perceived value and customer satisfaction.
  3. C) No, because customer satisfaction depends on customer expectations as well as product performance.
  4. D) No, because customers are not willing to sacrifice quality for lower prices.
  5. E) No, because value is different from customer-perceived value, and marketers can increase customers’ perceived value without changing the product’s actual performance.

Answer: C

Explanation: C) Value and satisfaction are different things. Customers can be dissatisfied even when they believe they are getting value if their expectations are high enough. So Choice C has it right. Choice A repeats the manager’s mistake. Choice B is true but irrelevant to the question at hand. Choice D is false, at least some of the time, and it’s also irrelevant. Choice E gets at the difference between value and customer-perceived value, but that still doesn’t explain why satisfaction can be low when perceived value is high. To explain that, you need to bring in customer expectations.

Page Ref: 184

Learning Outcome: Explain the entrepreneurship process

Difficulty: Difficult

Objective: 6.5 How can I avoid failure and keep my business healthy?

Classification: Critical Thinking

AACSB: Reflective Thinking Skills

A new portable technology, called Augmented Reality, offers a variety of services through a combination of wireless and camera phone technology. The camera in the handheld Augmented Reality device scans the surroundings, looking for images it recognizes. For example, when it sees a restaurant it recognizes, it calls up the restaurant’s menu as well as any reviews it has on file. It can even recognize faces and associate them with names and phone numbers. Makers of the technology predict that it will be especially attractive to people who grew up using the Web to conduct research, and so they predict a very high adoption rate.

124) Which of the following, if true, supports the makers’ prediction regarding the adoption rate of Augmented Reality?

  1. A) The interface for Augmented Reality strongly resembles those of most Web browsers.
  2. B) Current smartphone technology allows users to browse the Web and find restaurant reviews.
  3. C) The cameras in most phones are less advanced than the cameras used by photography professionals.
  4. D) If sales are successful, the makers of Augmented Reality plan on developing a version that fits onto sunglasses.
  5. E) The Augmented Reality database is likely to include existing reviews that are available elsewhere.

Answer: A

Explanation: A) Complexity is an important factor when it comes to the adoption rate of innovative products. If Choice A were true, then most potential users would be familiar with the basic interface of Augmented Reality, which makes a high adoption rate more likely. Choices B and E undermine the argument by suggesting that Augmented Reality is not so innovative after all. Choice C is irrelevant because Augmented Reality is not selling against high-end cameras. Choice D changes the subject to the next product, but the issue at hand is Augmented Reality.

Page Ref: 184

Learning Outcome: Explain the entrepreneurship process

Difficulty: Moderate

Objective: 6.5 How can I avoid failure and keep my business healthy?

Classification: Critical Thinking

AACSB: Reflective Thinking Skills

125) Which of the following, if true, weakens the makers’ prediction regarding the adoption rate of Augmented Reality?

  1. A) The percentage of people that have used Web browsers is higher than the percentage that have used camera phones.
  2. B) Augmented Reality would have been impossible to produce had camera phones not been invented.
  3. C) No other device is capable of automatically scanning the outside environment and forming connections with a Web-based database.
  4. D) Using Augmented Reality requires the purchase of an expensive handheld device as well as a monthly subscription.
  5. E) Augmented Reality licenses search technology from the leading producer of search engines.

Answer: D

Explanation: D) High expense is a barrier to high adoption rates. So Choice D makes a high adoption rate less likely. Choice A: Augmented Reality springs from browsers and from cameras, but there’s nothing special to be learned from the relative popularity of each. Choice B is irrelevant because camera phones were invented. Choice C strengthens the argument by making Augmented Reality sound more innovative. Choice E could strengthen the argument only by suggesting that the search technology would be pretty good.

Page Ref: 184

Learning Outcome: Explain the entrepreneurship process

Difficulty: Moderate

Objective: 6.5 How can I avoid failure and keep my business healthy?

Classification: Critical Thinking

AACSB: Reflective Thinking Skills

Business: A Practical Introduction (Williams/Sawyer/Berston)

Chapter 7 Management and Leadership

1) Efficiency focuses on the means of achieving goals, whereas effectiveness focuses on the results themselves.

Answer: TRUE

Explanation: To be efficient is to make use of resources in a wise and cost-effective manner while in the process of attaining goals. To be effective is to achieve the goals.

Page Ref: 192

Learning Outcome: Describe the skills and functions of management

Difficulty: Easy

Objective: 7.1 Why are managers needed, what do they do, what are their levels, and how do they make decisions?

Classification: Concept

2) To be effective means to use resources wisely and cost-effectively.

Answer: FALSE

Explanation: To use people, money, raw materials, and other resources wisely and cost-effectively is the definition of being efficient.

Page Ref: 192

Learning Outcome: Describe the skills and functions of management

Difficulty: Easy

Objective: 7.1 Why are managers needed, what do they do, what are their levels, and how do they make decisions?

Classification: Concept

3) One of a manager’s responsibilities is motivating people to work hard to achieve the organization’s goals.

Answer: TRUE

Explanation: Motivating the people under and around you is one of the roles of a leader. “Leader” and “manager” are not the same thing, but leading is one of the functions expected of a manager.

Page Ref: 193

Learning Outcome: Discuss the factors that affect motivation and behavior in the workplace

Difficulty: Easy

Objective: 7.1 Why are managers needed, what do they do, what are their levels, and how do they make decisions?

Classification: Concept

4) Middle managers determine the overall direction of the organization and establish the objectives, strategies, and policies for it.

Answer: FALSE

Explanation: Top management makes the long-term decisions about the overall direction of the organization and establishes objectives, strategies, and policies for it.

Page Ref: 194

Learning Outcome: Describe the skills and functions of management

Difficulty: Easy

Objective: 7.1 Why are managers needed, what do they do, what are their levels, and how do they make decisions?

Classification: Concept

5) Bernardo Rodriguez is the chief operating officer of a small IT firm operating in the northeastern United States. He and the chief executive officer feel it is time to decide whether to expand to other regions, so the next best step for them will be to isolate themselves and make the decision.

Answer: FALSE

Explanation: The COO and CEO at this point are in the second step of decision making. They should seek the opinions of others and brainstorm ideas of what expansion would entail.

Page Ref: 195

Learning Outcome: Describe the skills and functions of management

Difficulty: Moderate

Objective: 7.1 Why are managers needed, what do they do, what are their levels, and how do they make decisions?

Classification: Application

AACSB: Communication Abilities

6) Fred King runs a small bookstore coffee shop. His newest employee has recommended a new, lower-cost vendor for the store’s coffee supplies. However, the store would have to pay this vendor in cash, and the vendor cannot guarantee the brands or flavors of coffee from month to month. Fred immediately feels uncomfortable with the deal and says he’ll pass. Fred is exhibiting good ethical judgment.

Answer: TRUE

Explanation: At times, an alternative will seem too good to be true. An ethical manager will question unusual business practices. A request for payment in cash, for example, could be an attempt by the vendor to bypass paying sales tax.

Page Ref: 196

Learning Outcome: Discuss the roles of ethics and corporate responsibility in business

Difficulty: Moderate

Objective: 7.1 Why are managers needed, what do they do, what are their levels, and how do they make decisions?

Classification: Application

7) When a manager is ignoring actual evidence and hanging on to prior beliefs, the manager is showing a prior-hypothesis bias.

Answer: TRUE

Explanation: Sometimes managers with prior experience will ignore evidence that contradicts their beliefs and continue to hang on to their old hypotheses.

Page Ref: 197

Learning Outcome: Discuss the factors that affect motivation and behavior in the workplace

Difficulty: Moderate

Objective: 7.1 Why are managers needed, what do they do, what are their levels, and how do they make decisions?

Classification: Concept

8) Strategic plans and operational plans can be just as effective when developed in the absence of a company’s mission and vision.

Answer: FALSE

Explanation: Operational plans might help an organization function on a week-to-week basis, but they still need to have the company’s goals in mind. An effective strategic plan needs to be developed with the company’s overarching purpose in mind.

Page Ref: 199

Learning Outcome: Summarize the major theories of and approaches to leadership

Difficulty: Easy

Objective: 7.2 What are the benefits of planning, and what is the planning process?

Classification: Concept

9) Collis P. Huntington, founder of Newport News Shipbuilding & Drydock Company, said, “We shall build good ships here: at a profit if we can, at a loss if we must, but always good ships.” This is an example of a strategic plan.

Answer: FALSE

Explanation: This is an example of a mission statement. Huntington expressed it more than 100 years ago and it has been used as inspiration by many others in the century since.

Page Ref: 199

Learning Outcome: Discuss the factors that affect motivation and behavior in the workplace

Difficulty: Moderate

Objective: 7.2 What are the benefits of planning, and what is the planning process?

Classification: Application

10) Good organizational planning starts at the grassroots level, where objectives and goals are conceived and then “bubbled up” to top management to be developed into strategic planning documents.

Answer: FALSE

Explanation: The planning process starts with clear vision and mission statements, from which top management develops strategic plans that are passed downward through the organization. Tactical plans, then operational plans and correlating goals and objectives, flow from these strategic documents.

Page Ref: 200

Learning Outcome: Discuss the factors that affect motivation and behavior in the workplace

Difficulty: Moderate

Objective: 7.2 What are the benefits of planning, and what is the planning process?

Classification: Concept

11) An objective is a specific, short-term target designed to achieve the broad, long-range targets that an organization wishes to attain.

Answer: TRUE

Explanation: An organization’s long-range targets are its goals. Objectives are developed to attain those goals.

Page Ref: 202

Learning Outcome: Discuss the factors that affect motivation and behavior in the workplace

Difficulty: Easy

Objective: 7.2 What are the benefits of planning, and what is the planning process?

Classification: Concept

12) Goals are broad, long-range targets that an organization wishes to attain.

Answer: TRUE

Explanation: Goals are set by all levels of management based on the higher-tiered planning documents. From them, objectives are then developed for shorter-range planning.

Page Ref: 202

Learning Outcome: Discuss the factors that affect motivation and behavior in the workplace

Difficulty: Easy

Objective: 7.2 What are the benefits of planning, and what is the planning process?

Classification: Concept

13) In a SWOT analysis, the strengths and weaknesses are external.

Answer: FALSE

Explanation: The strengths and weaknesses are internal, while the opportunities and threats are external.

Page Ref: 202

Learning Outcome: Summarize the major theories of and approaches to leadership

Difficulty: Easy

Objective: 7.2 What are the benefits of planning, and what is the planning process?

Classification: Concept

14) If Suzy’s Donuts conducted a SWOT analysis, Suzy’s would identify a new Dunkin’ Donuts across the street as a threat.

Answer: TRUE

Explanation: Competition so close with overlapping product lines is definitely a threat.

Page Ref: 202

Learning Outcome: Summarize the major theories of and approaches to leadership

Difficulty: Easy

Objective: 7.2 What are the benefits of planning, and what is the planning process?

Classification: Application

15) Contingency plans can deal with emergencies or just uncertainty, so they should look at environmental, economic, competitive, and budgetary concerns, for example.

Answer: TRUE

Explanation: Contingency plans are alternative plans to use in the event that the original plans are unworkable or emergency plans to use in case of disaster. They offer a good way to get managers to think strategically.

Page Ref: 203

Learning Outcome: Discuss the factors that affect motivation and behavior in the workplace

Difficulty: Moderate

Objective: 7.2 What are the benefits of planning, and what is the planning process?

Classification: Concept

16) Hierarchical organizations are frequently represented in an organization chart.

Answer: TRUE

Explanation: Hierarchical organizations are also referred to as “vertical” or “pyramid.” This organizational style is a traditional form of management and can be successful. Lines of authority are very clear.

Page Ref: 204

Learning Outcome: Discuss the factors that influence decisions about organizational structure

Difficulty: Easy

Objective: 7.3 What do organization charts show, and what three skills do managers need?

Classification: Concept

17) Of technical, human, and conceptual skills, technical skills are often the most important for supervisory managers to have.

Answer: TRUE

Explanation: Supervisory managers often train employees on technical skills, or have to explain technical content to them. It helps to have a good grounding in the subject.

Page Ref: 205

Learning Outcome: Describe the skills and functions of management

Difficulty: Easy

Objective: 7.3 What do organization charts show, and what three skills do managers need?

Classification: Concept

18) Top managers do not need human skills.

Answer: FALSE

Explanation: All three levels of management need human skills so they can motivate, inspire trust in, and communicate with others.

Page Ref: 205

Learning Outcome: Describe the skills and functions of management

Difficulty: Easy

Objective: 7.3 What do organization charts show, and what three skills do managers need?

Classification: Concept

19) Dona has just been promoted to a supervisory management position, but her manager, Tom, has received feedback that her human skills are lacking. He should encourage Dona to attend a one-day training course that guarantees improvement in human skills because they are easy skills to learn.

Answer: FALSE

Explanation: Human skills are the set of skills that enable people to work cooperatively with others. They are the hardest skills to master. A short training course that guarantees improvement should be evaluated with skepticism; a mentoring program might work better.

Page Ref: 205

Learning Outcome: Describe the skills and functions of management

Difficulty: Moderate

Objective: 7.3 What do organization charts show, and what three skills do managers need?

Classification: Application

AACSB: Communication Abilities

20) TJ has an equal mix of technical, human, and conceptual skills. He is currently a supervisory manager but hopes to get a promotion. Having this mix of skills guarantees that he will become a middle manager.

Answer: FALSE

Explanation: Middle managers need to have an equal mix of the three skills. TJ is off to a good start, but there are no guarantees of promotions in business.

Page Ref: 206

Learning Outcome: Describe the skills and functions of management

Difficulty: Moderate

Objective: 7.3 What do organization charts show, and what three skills do managers need?

Classification: Application

21) Leaders cope with change in part by communicating the new direction so that employees can align behind it.

Answer: TRUE

Explanation: Successful businesses need leaders who can deal with great changes by creating a vision and strategic plan and inspiring others to rally around common goals.

Page Ref: 207

Learning Outcome: Describe the skills and functions of management

Difficulty: Easy

Objective: 7.4 How do leaders and managers differ, and what are the different types of leaders?

Classification: Concept

AACSB: Communication Abilities

22) One way that managers cope with complexity is by controlling and problem solving, monitoring results, and solving problems as they arise.

Answer: TRUE

Explanation: The other ways are by planning and budgeting, setting targets and specifying the resources and means for achieving them; and by organizing, creating the necessary structure and hiring the people to fulfill the jobs.

Page Ref: 208

Learning Outcome: Describe the skills and functions of management

Difficulty: Easy

Objective: 7.4 How do leaders and managers differ, and what are the different types of leaders?

Classification: Concept

23) You can be a leader without being a manager.

Answer: TRUE

Explanation: Nonmanagement employees can motivate and inspire others, which is part of the definition of being a leader. Unfortunately, you can also be a manager without being a leader.

Page Ref: 208

Learning Outcome: Describe the skills and functions of management

Difficulty: Easy

Objective: 7.4 How do leaders and managers differ, and what are the different types of leaders?

Classification: Concept

AACSB: Communication Abilities

24) Autocratic leadership is a thing of the past and has no current successful example.

Answer: FALSE

Explanation: The U.S. military branches still very much follow an autocratic leadership model, as do Martha Stewart and Donald Trump.

Page Ref: 208

Learning Outcome: Summarize the major theories of and approaches to leadership

Difficulty: Easy

Objective: 7.4 How do leaders and managers differ, and what are the different types of leaders?

Classification: Concept

25) Physicians On Call is a new practice of medical doctors who cater to wealthy elderly clients by making house calls and always accepting phone calls. One of the physicians is considered the manager of the practice, but generally decisions are made through discussion and consensus. This is an example of free-rein leadership.

Answer: TRUE

Explanation: Professionals such as research scientists, physicians, and computer engineers often respond well in an environment of free-rein or laissez-faire leadership.

Page Ref: 209

Learning Outcome: Summarize the major theories of and approaches to leadership

Difficulty: Easy

Objective: 7.4 How do leaders and managers differ, and what are the different types of leaders?

Classification: Application

26) Transactional leaders are concerned with setting goals, clarifying employee roles, providing rewards and punishments related to performance, and monitoring progress.

Answer: TRUE

Explanation: Transactional leadership focuses on creating a smooth-running organization, motivating employees to meet performance goals.

Page Ref: 210

Learning Outcome: Summarize the major theories of and approaches to leadership

Difficulty: Moderate

Objective: 7.4 How do leaders and managers differ, and what are the different types of leaders?

Classification: Concept

27) Transformational leaders model desirable values by displaying high ethical standards.

Answer: TRUE

Explanation: Transformational leaders inspire trust by expressing their integrity.

Page Ref: 211

Learning Outcome: Summarize the major theories of and approaches to leadership

Difficulty: Moderate

Objective: 7.4 How do leaders and managers differ, and what are the different types of leaders?

Classification: Concept

28) Control, or monitoring performance, is a way of answering the question “Are we on track?”

Answer: TRUE

Explanation: Control entails comparing actual performance with goals, and taking corrective action as needed. Making sure that performance meets goals means being concerned with achieving productivity and realizing results.

Page Ref: 213

Learning Outcome: Discuss the factors that affect motivation and behavior in the workplace

Difficulty: Easy

Objective: 7.5 How does control work, and how should I use it to be an effective manager?

Classification: Concept

29) A control standard is the minimum acceptable performance level.

Answer: FALSE

Explanation: A control standard is the desired performance level for a given goal.

Page Ref: 215

Learning Outcome: Discuss the factors that affect motivation and behavior in the workplace

Difficulty: Moderate

Objective: 7.5 How does control work, and how should I use it to be an effective manager?

Classification: Concept

30) MBWA, or management by walking around, does no good and only makes most employees nervous.

Answer: FALSE

Explanation: MBWA is an accepted best practice for keeping lines of communication open between levels of management and workers and for spotting problem areas as well as opportunities. A minority of employees see it as something negative.

Page Ref: 216

Learning Outcome: Discuss the factors that affect motivation and behavior in the workplace

Difficulty: Easy

Objective: 7.5 How does control work, and how should I use it to be an effective manager?

Classification: Application

31) Negative performance should be corrected by examining the reasons why performance is less than desirable and taking appropriate action to achieve desired outcomes.

Answer: TRUE

Explanation: Sometimes performance standards are unrealistic and need to be altered. Sometimes employees haven’t been given the resources for achieving the standards. And sometimes employees may need more attention and direction from management.

Page Ref: 216

Learning Outcome: Discuss the factors that affect motivation and behavior in the workplace

Difficulty: Easy

Objective: 7.5 How does control work, and how should I use it to be an effective manager?

Classification: Concept

32) Which of the following functions is one of the major functions of being a middle manager?

  1. A) implementing operational policies
  2. B) establishing organizational strategies
  3. C) making operational policies
  4. D) making organizational decisions
  5. E) hiring new employees

Answer: A

Explanation: A) Implementing policies and plans of top managers is a main function of a middle manager.

Page Ref: 195

Learning Outcome: Describe the skills and functions of management

Difficulty: Moderate

Objective: 7.1 Why are managers needed, what do they do, what are their levels, and how do they make decisions?

Classification: Concept

33) Which of the following activities is a part of the planning function of management?

  1. A) measuring results against goals
  2. B) allocating resources
  3. C) developing strategy
  4. D) creating organizational structure
  5. E) taking corrective action

Answer: C

Explanation: C) Planning is defined as setting goals and deciding how to achieve them. Top executives must develop the strategies to achieve these goals.

Page Ref: 193

Learning Outcome: Describe the skills and functions of management

Difficulty: Moderate

Objective: 7.1 Why are managers needed, what do they do, what are their levels, and how do they make decisions?

Classification: Concept

34) Which of the following activities is associated with the organizing phase of management?

  1. A) taking corrective action
  2. B) motivating employees
  3. C) monitoring performance
  4. D) allocating resources
  5. E) setting goals

Answer: D

Explanation: D) Organizing is defined as arranging tasks, people, and other resources to accomplish the work.

Page Ref: 193

Learning Outcome: Describe the skills and functions of management

Difficulty: Moderate

Objective: 7.1 Why are managers needed, what do they do, what are their levels, and how do they make decisions?

Classification: Concept

35) Which of the following is NOT a function that effective managers typically perform?

  1. A) motivating people through leadership
  2. B) organizing resources to accomplish goals
  3. C) performing everyday production tasks
  4. D) planning for goal achievement
  5. E) controlling the organization’s resources

Answer: C

Explanation: C) Everyday production tasks are generally handled by the company’s nonmanagerial personnel.

Page Ref: 193-194

Learning Outcome: Describe the skills and functions of management

Difficulty: Moderate

Objective: 7.1 Why are managers needed, what do they do, what are their levels, and how do they make decisions?

Classification: Concept

36) Ursula M. Burns, who was named CEO of Xerox in 2009, has been commended for taking the company in new directions by pushing products with a new vision. Which of the primary management functions is Burns displaying?

  1. A) marketing
  2. B) leading
  3. C) planning
  4. D) organizing
  5. E) monitoring

Answer: B

Explanation: B) Burns steers the company in a new direction with strong leadership.

Page Ref: 193

Learning Outcome: Describe the skills and functions of management

Difficulty: Moderate

Objective: 7.1 Why are managers needed, what do they do, what are their levels, and how do they make decisions?

Classification: Application

37) Kim has recently been promoted to a position as a senior vice president in her organization. Which of the following, if true, most supports Kim being a good fit for top management?

  1. A) She knows how to focus in a disorganized work environment.
  2. B) She is personable at social events held by other managers within the organization.
  3. C) She can see the “big picture” when deciding whether to act on a new market opportunity.
  4. D) She is able to spend more time with employees at work than with family and friends at home.
  5. E) She knows how to be assertive enough to get the attention of a large number of people.

Answer: C

Explanation: C) Top managers make long-term decisions about the overall direction of the organization.

Page Ref: 194

Learning Outcome: Describe the skills and functions of management

Difficulty: Moderate

Objective: 7.1 Why are managers needed, what do they do, what are their levels, and how do they make decisions?

Classification: Application

AACSB: Analytic Skills

38) Danielle’s responsibility as a manager of a large corporation is to oversee staff whose duties include ordering office supplies, scheduling meetings, and planning events. What type of manager is Danielle?

  1. A) a chief operations officer
  2. B) a top manager
  3. C) a middle manager
  4. D) a supervisory manager
  5. E) a division manager

Answer: D

Explanation: D) A supervisory manager supervises employees who carry out specific, day-to-day tasks for an organization.

Page Ref: 195

Learning Outcome: Describe the skills and functions of management

Difficulty: Moderate

Objective: 7.1 Why are managers needed, what do they do, what are their levels, and how do they make decisions?

Classification: Application

39) Which of the following activities is an important part of controlling?

  1. A) establishing vision
  2. B) setting organizational goals
  3. C) measuring performance
  4. D) changing organizational structure
  5. E) hiring additional personnel

Answer: C

Explanation: C) The very definition of controlling is monitoring performance, comparing it with goals, and taking corrective action as needed. An important part of monitoring performance is measuring it.

Page Ref: 194

Learning Outcome: Discuss the factors that affect motivation and behavior in the workplace

Difficulty: Easy

Objective: 7.1 Why are managers needed, what do they do, what are their levels, and how do they make decisions?

Classification: Concept

40) What is the first step in practical decision making?

  1. A) Dream up possible solutions.
  2. B) Weigh alternative solutions and select one.
  3. C) Identify the problem or opportunity.
  4. D) Implement the solution, then evaluate it.
  5. E) Call a meeting.

Answer: C

Explanation: C) Before the problem is understood, efforts at resolving it might waste resources and time. Creative managers also often find opportunities—favorable circumstances that present possibilities for progress beyond existing goals.

Page Ref: 195

Learning Outcome: Discuss the factors that affect motivation and behavior in the workplace

Difficulty: Easy

Objective: 7.1 Why are managers needed, what do they do, what are their levels, and how do they make decisions?

Classification: Concept

41) The Salter sisters, CEO and COO of Salter Sisters Sweets, have asked their operations director for his reaction to their latest idea: a marshmallow snowman with a chocolate center that melts into a gooey, messy puddle to be scooped up by animal-shaped cookies. The operations director thinks that when melted too long, marshmallow hardens and sticks to the plate. He also thinks that parents won’t like the cleanup involved with the product. What should the Smith sisters do next?

  1. A) market only the animal-shaped cookies
  2. B) market the product without the marshmallow
  3. C) manufacture the product and see how it sells
  4. D) do more product research and survey their customers
  5. E) defer to the Operations Director and pass on the idea

Answer: D

Explanation: D) This is a time when the company would be wise to follow the steps of practical decision making, starting with identifying what the problem really is. That is, does the marshmallow harden when melted and will parents not like the product? Product research and surveying potential customers is the wisest choice of action.

Page Ref: 195-197

Learning Outcome: Discuss the factors that affect motivation and behavior in the workplace

Difficulty: Difficult

Objective: 7.1 Why are managers needed, what do they do, what are their levels, and how do they make decisions?

Classification: Application

AACSB: Reflective Thinking Skills

42) How might an effective manager best choose a plan of action when making a decision?

  1. A) by generating a list of pros and cons and choosing based on the list
  2. B) by using the results of a survey that was sent to key customers who will be affected
  3. C) by sending a feedback form to customers after the plan has been executed
  4. D) by creating only one plan with no alternatives so it has to work
  5. E) by reviewing what other managers have done in the same situation and doing the same

Answer: B

Explanation: B) An effective manager will consider the effects on customers before taking a certain action. Any proposed solution has to be evaluated under current circumstances.

Page Ref: 195

Learning Outcome: Describe the skills and functions of management

Difficulty: Moderate

Objective: 7.1 Why are managers needed, what do they do, what are their levels, and how do they make decisions?

Classification: Concept

AACSB: Reflective Thinking Skills

43) A sales team has been regularly convening to brainstorm better ways to reach out to customers. After a few months, they have come up with only one or two workable ideas. Their manager is wondering what is behind their slow progress. Which of the following is the most likely reason for the team’s slow progress in brainstorming?

  1. A) The desire not to stand out in the group makes people reluctant to share ideas.
  2. B) Brainstorming is usually not an effective way to gather ideas.
  3. C) The team wants to make a decision that guarantees a positive outcome.
  4. D) People are having a hard time focusing without their manager to guide them.
  5. E) The team is stuck in the information gathering phase of the assignment.

Answer: A

Explanation: A) This phenomenon can hamper the sharing of creative ideas in a group setting.

Page Ref: 197

Learning Outcome: Discuss the factors that affect motivation and behavior in the workplace

Difficulty: Moderate

Objective: 7.1 Why are managers needed, what do they do, what are their levels, and how do they make decisions?

Classification: Application

AACSB: Reflective Thinking Skills

44) People who are from similar backgrounds and from similar sectors of the company tend to have a set of familiar ideas and work with the same set of unspoken assumptions. This mindset may lead to rejecting different ideas without fair examination. What is this phenomenon called?

  1. A) ignoring randomness bias
  2. B) overconfidence bias
  3. C) analysis paralysis
  4. D) group think
  5. E) prior-hypothesis bias

Answer: D

Explanation: D) Group think, or uncritical thinking, can be disastrous in brainstorming sessions.

Page Ref: 197

Learning Outcome: Discuss the factors that affect motivation and behavior in the workplace

Difficulty: Moderate

Objective: 7.1 Why are managers needed, what do they do, what are their levels, and how do they make decisions?

Classification: Concept

45) Which of the following statements is often true of a team characterized by group think?

  1. A) Team members come from different sectors of the company.
  2. B) Team members use a broad-minded approach when dealing with decisions and problems.
  3. C) Team members consider different ideas at great length before taking action.
  4. D) Team members have a set of familiar ideas and work with the same set of unspoken assumptions.
  5. E) Team members come from diverse educational and class backgrounds.

Answer: D

Explanation: D) Diversity in a group does not guarantee critical thinking but reduces the likelihood of group think.

Page Ref: 197

Learning Outcome: Discuss the factors that affect motivation and behavior in the workplace

Difficulty: Difficult

Objective: 7.1 Why are managers needed, what do they do, what are their levels, and how do they make decisions?

Classification: Concept

46) Javier has had great success as a manager, leading the information technology department of a large corporation. His success has been attributed to his ability to create clear tactical plans that aid in executing the organization’s strategic vision. What type of manager is Javier?

  1. A) a chief executive officer
  2. B) a top manager
  3. C) a middle manager
  4. D) a supervisory manager
  5. E) a project leader

Answer: C

Explanation: C) To be successful, middle managers must have a strong strategic vision coming down from top managers that can be executed through tactical planning.

Page Ref: 200

Learning Outcome: Describe the skills and functions of management

Difficulty: Moderate

Objective: 7.1 Why are managers needed, what do they do, what are their levels, and how do they make decisions?

Classification: Application

47) Cody is being considered for promotion from middle management to top management. He is unsure whether he is qualified for the position. He wants to ensure that he is an effective manager and not simply having success in the workplace because of his multiple promotions in the last few years. Although he’s never been a top manager, other members of the management team think there are some things he does in his personal life that qualify him for the position. What personal activity is management most likely think is a good indicator that Cody will be an effective top manager?

  1. A) He is part of a theatrical group that gives performances to hundreds of people, showing that he is not afraid to speak in front of a crowd.
  2. B) He juggles being a parent, working as a middle manager, and having personal hobbies, showing an ability to balance many aspects of his life.
  3. C) He has many siblings, showing that he knows how to share space with a large number of people.
  4. D) He quickly learns how to play new games by following the rules explained to him.
  5. E) He was a member of the debate team in high school and consequently is well-versed in being able to argue both sides of an issue.

Answer: B

Explanation: B) This scenario shows Cody’s personal skills of time-management and interpersonal skills that will contribute to him being an effective top manager.

Page Ref: 205

Learning Outcome: Describe the skills and functions of management

Difficulty: Moderate

Objective: 7.1 Why are managers needed, what do they do, what are their levels, and how do they make decisions?

Classification: Application

AACSB: Reflective Thinking Skills

48) When conducting a SWOT analysis of an organization, which question can help management brainstorm for the strengths section of the analysis?

  1. A) What external changes present interesting possibilities for new products or markets?
  2. B) What potential actions by competitors pose the biggest challenge?
  3. C) What necessary skills do the organization’s employees currently lack?
  4. D) What does the organization offer that makes it stand out from the other organizations?
  5. E) What external economic forces can affect the organization’s bottom line?

Answer: D

Explanation: D) This question helps management brainstorm and identify its internal strengths.

Page Ref: 202

Learning Outcome: Discuss the factors that affect motivation and behavior in the workplace

Difficulty: Easy

Objective: 7.2 What are the benefits of planning, and what is the planning process?

Classification: Concept

49) Which one of these is NOT a primary benefit of planning?

  1. A) It helps a company cope with uncertainty.
  2. B) It helps a company keep employee morale high.
  3. C) It helps a company coordinate activities.
  4. D) It helps a company check on its progress.
  5. E) It helps a company think ahead.

Answer: B

Explanation: B) Planning doesn’t necessarily directly affect morale. The purpose of planning is to help a company think ahead.

Page Ref: 199

Learning Outcome: Discuss the factors that affect motivation and behavior in the workplace

Difficulty: Moderate

Objective: 7.2 What are the benefits of planning, and what is the planning process?

Classification: Concept

50) Which one of the following is NOT a benefit of planning?

  1. A) Planning helps you cope with uncertainty.
  2. B) Planning helps you avoid unforeseen emergencies.
  3. C) Planning helps you think ahead.
  4. D) Planning helps you coordinate activities.
  5. E) Planning helps you check on your progress.

Answer: B

Explanation: B) Planning can help you deal with or recover from emergencies, but it cannot help you avoid unforeseen emergencies.

Page Ref: 199

Learning Outcome: Summarize the major theories of and approaches to leadership

Difficulty: Moderate

Objective: 7.2 What are the benefits of planning, and what is the planning process?

Classification: Concept

51) What do a company’s vision and mission statement have in common?

  1. A) They are both used to generate profits.
  2. B) They are both directed toward employees.
  3. C) They are both used to keep management on track.
  4. D) They are both directed toward customers.
  5. E) They are both used to set short-term objectives.

Answer: C

Explanation: C) A company’s vision and mission statement are used to keep management moving toward defined goals.

Page Ref: 199

Learning Outcome: Summarize the major theories of and approaches to leadership

Difficulty: Moderate

Objective: 7.2 What are the benefits of planning, and what is the planning process?

Classification: Concept

52) What is the major difference between a vision and a mission statement?

  1. A) A vision statement is an operational statement, whereas a mission statement is a strategic concept.
  2. B) Unlike a mission statement, a vision statement discusses the core values of the organization.
  3. C) A vision statement is a current description of purpose, whereas a mission statement is a futuristic projection.
  4. D) Unlike a vision statement, which is directed internally, a mission statement has the customer in mind.
  5. E) A mission statement, which is directed internally, has only the company in mind.

Answer: D

Explanation: D) A vision statement is directed internally as a statement of where the company wants to be in the future given the context of the world and the company’s competition. A company’s mission statement can be seen as a top-tier marketing document as it describes the company’s purpose for existing now, its goods and services, and what it can provide.

Page Ref: 199

Learning Outcome: Summarize the major theories of and approaches to leadership

Difficulty: Moderate

Objective: 7.2 What are the benefits of planning, and what is the planning process?

Classification: Concept

53) Which of the following BEST describes a well-thought-out strategic plan?

  1. A) The strategic plan depends on daily or weekly schedules and focuses on specific departments or employees.
  2. B) The strategic plan lays out what contributions each work unit can make over the next several years.
  3. C) The strategic plan presents an analysis of current strengths, weaknesses, and anticipated changes within an organization.
  4. D) The strategic plan reflects what is going on inside and outside the organization and how those conditions will affect the organization in the future.
  5. E) The strategic plan provides a current description of the organization’s purpose, basic goals, and philosophies.

Answer: D

Explanation: D) The strategic plan needs to be developed with an eye to global markets, the economy, and potential competition, as well as anticipated internal and external resources. Thinking ahead about these uncertainties helps top management plan current as well as future actions.

Page Ref: 200

Learning Outcome: Summarize the major theories of and approaches to leadership

Difficulty: Moderate

Objective: 7.2 What are the benefits of planning, and what is the planning process?

Classification: Concept

54) Top managers look ahead to the next one to five years in which of the following type of plan?

  1. A) strategic
  2. B) vision
  3. C) operational
  4. D) tactical
  5. E) mission

Answer: A

Explanation: A) With the company’s vision and mission in mind, top managers like CEOs look at long-term goals set according to the resources they anticipate having.

Page Ref: 200

Learning Outcome: Summarize the major theories of and approaches to leadership

Difficulty: Easy

Objective: 7.2 What are the benefits of planning, and what is the planning process?

Classification: Concept

55) The people involved with creating a strategic plan must incorporate each of the following EXCEPT which one?

  1. A) fluctuations in the economy
  2. B) organizational resources
  3. C) long-term goals
  4. D) personal bias
  5. E) short-term objectives

Answer: D

Explanation: D) Personal bias has no place in any of the planning documents.

Page Ref: 200

Learning Outcome: Summarize the major theories of and approaches to leadership

Difficulty: Moderate

Objective: 7.2 What are the benefits of planning, and what is the planning process?

Classification: Concept

56) Which of the following tasks is usually performed by top-level managers?

  1. A) generating strategic plans
  2. B) coordinating specific plans with the established strategic vision
  3. C) establishing short-term goals
  4. D) carrying out operational planning
  5. E) verifying that objectives are established for all goals

Answer: A

Explanation: A) Top managers generate strategic plans looking at the next one to five years based on the resources they anticipate having, with visionary and directional thinking. Middle managers implement strategic plans.

Page Ref: 200

Learning Outcome: Describe the skills and functions of management

Difficulty: Easy

Objective: 7.2 What are the benefits of planning, and what is the planning process?

Classification: Concept

57) Who are responsible for tactical planning and creating specific plans that coordinate with the firm’s strategic vision?

  1. A) strategic managers
  2. B) middle managers
  3. C) supervisory managers
  4. D) top managers
  5. E) CEOs

Answer: B

Explanation: B) Middle managers take the strategic plans of top management and apply their knowledge of their units’ specific resources and capabilities to assess what is achievable over the next 6 to 24 months.

Page Ref: 200

Learning Outcome: Describe the skills and functions of management

Difficulty: Easy

Objective: 7.2 What are the benefits of planning, and what is the planning process?

Classification: Concept

58) How might a tactical plan contribute to a company’s success?

  1. A) It depends on daily or weekly schedules and focuses on specific departments or employees.
  2. B) It determines the resources and actions required to implement particular aspects of the organization’s course of action for a 6- to 24-month time frame.
  3. C) It presents an analysis of strengths, weaknesses, and anticipated changes within an organization for a 1- to 5-year time frame.
  4. D) It reflects what is going on inside and outside the organization and how those conditions and changes will affect the organization in the future.
  5. E) It provides a current description of the organization’s purpose, basic goals, and philosophies.

Answer: B

Explanation: B) Tactical plans are developed by middle managers with the input of both the supervisory managers reporting to them and top management. Tactical plans take the strategic plans of top management and translate them into actionable working plans for middle management and their units.

Page Ref: 200

Learning Outcome: Discuss the factors that affect motivation and behavior in the workplace

Difficulty: Moderate

Objective: 7.2 What are the benefits of planning, and what is the planning process?

Classification: Concept

59) Supervisory managers direct the tasks of nonmanagerial employees. Which of the following is true about the supervisors’ operational plans?

  1. A) They change on a daily basis.
  2. B) They look ahead one week to one year.
  3. C) They must be approved by top managers.
  4. D) They stem directly from the company’s vision.
  5. E) They bypass the company’s mission.

Answer: B

Explanation: B) Supervisory managers’ plans are operational documents based on the tactical plans of middle management. They might be very formulaic or they might require individual judgment to respond to unique situations.

Page Ref: 201

Learning Outcome: Describe the skills and functions of management

Difficulty: Easy

Objective: 7.2 What are the benefits of planning, and what is the planning process?

Classification: Concept

60) How does an operational plan contribute to a company’s success?

  1. A) It depends on daily or weekly schedules and focuses on specific departments or employees.
  2. B) It determines the resources and the actions required to implement particular aspects of the organization’s main course of action for a 6-month to a 2-year time frame.
  3. C) It presents an analysis of strengths, weaknesses, and anticipated changes within an organization for up to 5 years.
  4. D) It reflects what is going on inside and outside the organization and how those conditions and changes will affect the organization in the future.
  5. E) It provides a current description of the organization’s purpose, basic goals, and philosophies.

Answer: A

Explanation: A) An operational plan is developed by supervisory management and focuses on the tasks of nonmanagerial personnel.

Page Ref: 201

Learning Outcome: Discuss the factors that affect motivation and behavior in the workplace

Difficulty: Moderate

Objective: 7.2 What are the benefits of planning, and what is the planning process?

Classification: Concept

61) Janice is a supervisory manager in an office. Which objective would Janice’s operational plan NOT contain?

  1. A) Write annual performance reviews for nonmanagerial employees.
  2. B) Request professional development training for clerks and data entry personnel.
  3. C) Take corrective action to address personnel problems between workers.
  4. D) Evaluate nonmanagerial employees for possible promotion.
  5. E) Revise the mission statement so it reflects what the company actually does.

Answer: E

Explanation: E) Janice evidently supervises office workers. It is unlikely that she alone would be given the task of revising the mission statement, which should already reflect what the company does.

Page Ref: 201

Learning Outcome: Describe the skills and functions of management

Difficulty: Moderate

Objective: 7.2 What are the benefits of planning, and what is the planning process?

Classification: Application

62) Which tool helps to determine the strategic fit between an organization’s distinctive internal capabilities and the external possibilities relative to the business and economic environments?

  1. A) gap analysis
  2. B) operational plan
  3. C) organizational chart
  4. D) SWOT analysis
  5. E) strategic plan

Answer: D

Explanation: D) A SWOT analysis assesses the organization’s internal strengths and weaknesses as juxtaposed with possible external opportunities and threats. The resulting picture should help top management develop realistic strategic planning documents.

Page Ref: 202

Learning Outcome: Summarize the major theories of and approaches to leadership

Difficulty: Moderate

Objective: 7.2 What are the benefits of planning, and what is the planning process?

Classification: Concept

63) When conducting a SWOT analysis of an organization, which question can help management brainstorm for the threats section of the analysis?

  1. A) What external changes present interesting possibilities?
  2. B) What aspect of the organization’s vision can be altered?
  3. C) What necessary skills do the organization’s employees lack?
  4. D) What does the organization offer that makes it stand out from other organizations?
  5. E) What external economic forces can affect the organization’s bottom line?

Answer: E

Explanation: E) This question will help management evaluate outside factors that may pose a financial threat to the company.

Page Ref: 202

Learning Outcome: Discuss the factors that affect motivation and behavior in the workplace

Difficulty: Difficult

Objective: 7.2 What are the benefits of planning, and what is the planning process?

Classification: Application

64) When conducting a SWOT analysis of an organization, which of the following is best identified as a weakness of a company?

  1. A) increased competition in the industry
  2. B) employees lacking necessary skills
  3. C) energy-reduction technology
  4. D) changing buyer tastes
  5. E) rise in utility costs

Answer: B

Explanation: B) If a company’s employees are lacking necessary skills, management will need to decide whether the solution requires training for its current employees or hiring additional or different employees with more skills. Either solution incurs costs for the company and might also add time to any project. All these are considerations that management will need to factor into its SWOT analysis.

Page Ref: 202

Learning Outcome: Summarize the major theories of and approaches to leadership

Difficulty: Moderate

Objective: 7.2 What are the benefits of planning, and what is the planning process?

Classification: Concept

65) When conducting a SWOT analysis of an organization, which question can help management brainstorm for the weaknesses section of the analysis?

  1. A) What external changes present interesting possibilities?
  2. B) What aspect of the organization’s vision can be altered?
  3. C) What necessary skills do the organization’s employees currently lack?
  4. D) What does the organization offer that makes it stand out from other organizations?
  5. E) What external economic forces can affect the organization’s bottom line?

Answer: C

Explanation: C) This question will help management brainstorm an organization’s internal weaknesses.

Page Ref: 202

Learning Outcome: Summarize the major theories of and approaches to leadership

Difficulty: Moderate

Objective: 7.2 What are the benefits of planning, and what is the planning process?

Classification: Application

66) When conducting a SWOT analysis of an organization, which question can help management brainstorm for the opportunities section of the analysis?

  1. A) What external changes present interesting possibilities?
  2. B) What aspect of the organization’s vision can be altered?
  3. C) What parts of the organization’s facilities need updating?
  4. D) What does the organization offer that makes it stand out from other organizations?
  5. E) What external economic forces can affect the organization’s bottom line?

Answer: A

Explanation: A) This question will help management brainstorm opportunities that external constituents may offer.

Page Ref: 202

Learning Outcome: Summarize the major theories of and approaches to leadership

Difficulty: Moderate

Objective: 7.2 What are the benefits of planning, and what is the planning process?

Classification: Application

67) A nation reduces the trade barriers on the export of vegetables. What does this represent for a vegetable exporter in that country?

  1. A) an internal strength
  2. B) an external opportunity
  3. C) an internal weakness
  4. D) an internal threat
  5. E) an external threat

Answer: B

Explanation: B) If the trade barriers are reduced, a vegetable exporter will be able to export more out of the country.

Page Ref: 202

Learning Outcome: Summarize the major theories of and approaches to leadership

Difficulty: Moderate

Objective: 7.2 What are the benefits of planning, and what is the planning process?

Classification: Concept

68) Which of the following types of planning has its main focus on handling of unforeseen events?

  1. A) tactical planning
  2. B) contingency planning
  3. C) operational planning
  4. D) strategic planning
  5. E) SWOT planning

Answer: B

Explanation: B) Contingency plans are alternative plans that are developed in case the original schema prove to be unworkable.

Page Ref: 203

Learning Outcome: Summarize the major theories of and approaches to leadership

Difficulty: Easy

Objective: 7.2 What are the benefits of planning, and what is the planning process?

Classification: Concept

69) Which of the following is NOT a characteristic of a solid contingency plan?

  1. A) contact information for an on-site crisis manager
  2. B) information systems backup software
  3. C) an evacuation plan
  4. D) a prioritized list of critical operations
  5. E) a summary of common types of emergencies

Answer: E

Explanation: E) A summary of common types of emergencies would not be of much assistance in a crisis. However, a good contingency plan will provide all of the other information listed in anticipation of an emergency situation.

Page Ref: 203

Learning Outcome: Summarize the major theories of and approaches to leadership

Difficulty: Moderate

Objective: 7.2 What are the benefits of planning, and what is the planning process?

Classification: Application

70) Which of the following would NOT be included in a Hazardous Material Contingency Plan?

  1. A) contact phone number for the emergency coordinator
  2. B) weekly schedules designating operational duties
  3. C) names of laboratory supervisors
  4. D) building evacuation procedures
  5. E) designation of chemical storage areas

Answer: B

Explanation: B) Weekly schedules would be in an operational plan, not in a contingency plan.

Page Ref: 203

Learning Outcome: Summarize the major theories of and approaches to leadership

Difficulty: Difficult

Objective: 7.2 What are the benefits of planning, and what is the planning process?

Classification: Application

71) A crisis manager of an information technology firm has identified high-traffic areas throughout the firm’s building that would be the best places to post evacuation routes. Where should the location of those evacuation routes be recorded?

  1. A) in the firm’s operational plan
  2. B) in the firm’s contingency plan
  3. C) in the firm’s strategic plan
  4. D) in the firm’s tactical plan
  5. E) in the firm’s backup plan

Answer: B

Explanation: B) The contingency plan should include information pertinent to handling a crisis.

Page Ref: 203

Learning Outcome: Summarize the major theories of and approaches to leadership

Difficulty: Moderate

Objective: 7.2 What are the benefits of planning, and what is the planning process?

Classification: Application

72) What is one disadvantage associated with having a vertical organizational structure rather than a horizontal organizational structure?

  1. A) It has no specified functional teams with specialized skills.
  2. B) It shows no clear point of authority for decision making.
  3. C) It indicates a push toward collaborative work.
  4. D) It represents a solid structure that cannot change.
  5. E) It is a fragmented representation of the organization.

Answer: A

Explanation: A) In a vertical structure, communication across functions can be hampered.

Page Ref: 205

Learning Outcome: Discuss the factors that influence decisions about organizational structure

Difficulty: Moderate

Objective: 7.3 What do organization charts show, and what three skills do managers need?

Classification: Concept

73) What is one advantage associated with having a horizontal organizational structure?

  1. A) It has no specified functional teams with specialized skills.
  2. B) It shows a clear point of authority for decision making.
  3. C) It indicates a push toward collaborative work.
  4. D) It represents a solid structure that cannot change.
  5. E) It is a fragmented representation of the organization.

Answer: C

Explanation: C) With fewer layers of management, employees tend to be on more of a level “playing field,” which encourages collaboration and cross-functional communication.

Page Ref: 205

Learning Outcome: Discuss the factors that influence decisions about organizational structure

Difficulty: Moderate

Objective: 7.3 What do organization charts show, and what three skills do managers need?

Classification: Concept

74) Nigel Williams is the CEO of a toy manufacturing company. He constantly reviews his business model to look for ways to improve operations. He recently learned that it would be less expensive for him to produce toys if he simply purchased each part from a different distributor, instead of making them at his factory, and had the parts shipped to his factory, where they would then be assembled. He also thinks that collaborating with independent firms will allow him more time for research and marketing, instead of production, to keep abreast of current trends. What type of organization is Williams considering?

  1. A) horizontal organization
  2. B) vertical organization
  3. C) inverted organization
  4. D) network organization
  5. E) matrix organization

Answer: A

Explanation: A) A horizontal organization lends itself to collaboration with independent firms.

Page Ref: 205

Learning Outcome: Discuss the factors that influence decisions about organizational structure

Difficulty: Moderate

Objective: 7.3 What do organization charts show, and what three skills do managers need?

Classification: Application

75) Which type of chart represents a company organized by specific functions, such as marketing, finance, purchasing, information technology, and human resources?

  1. A) vertical
  2. B) social
  3. C) matrix
  4. D) horizontal
  5. E) web

Answer: A

Explanation: A) In a vertical organization chart, the vertical lines reflect the lines of authority with personnel grouped in the organization by similar job functions.

Page Ref: 205

Learning Outcome: Discuss the factors that influence decisions about organizational structure

Difficulty: Moderate

Objective: 7.3 What do organization charts show, and what three skills do managers need?

Classification: Concept

76) Which of the following is an advantage associated with a vertical organization?

  1. A) A vertical organization can respond quickly to changes in a market.
  2. B) Lines of communication are shorter in vertical organizations.
  3. C) Vertical organizations have a clear point of authority for decisions.
  4. D) Bureaucratic procedures do not delay decision making in vertical organizations.
  5. E) All employees are on equal footing in vertical organizations.

Answer: C

Explanation: C) In a vertical organization, the authority for each function is at the top.

Page Ref: 205

Learning Outcome: Discuss the factors that influence decisions about organizational structure

Difficulty: Easy

Objective: 7.3 What do organization charts show, and what three skills do managers need?

Classification: Concept

77) The company for which Rosalinda works is changing from a vertical hierarchy to a horizontal organizational structure. Which of the following considerations should Rosalinda expect NOT to see in the new structure?

  1. A) organization around cross-functional teams
  2. B) increased emphasis on multiple competencies
  3. C) increased emphasis on specialized expertise
  4. D) decreased emphasis on hierarchy
  5. E) decreased emphasis on teamwork

Answer: C

Explanation: C) A horizontal structure focuses on teams and employees with multiple competencies.

Page Ref: 205

Learning Outcome: Discuss the factors that influence decisions about organizational structure

Difficulty: Moderate

Objective: 7.3 What do organization charts show, and what three skills do managers need?

Classification: Application

78) Company ABC and Company XYZ are preparing for a merger but the two currently have very different organizational structures. Consequently, Lance has been hired as a consultant to reorganize the newly formed company. He envisions the project-based company as having employees grouped together based on their function, with the possibility of being accountable to both a department supervisor and a project manager. A large number of team members with specialized knowledge will report to a small, centralized group of top managers. What type of organizational structure best matches Lance’s vision?

  1. A) flat
  2. B) matrix
  3. C) horizontal
  4. D) vertical
  5. E) inverted

Answer: D

Explanation: D) A vertical organizational structure has multiple authority figures in charge and utilizes employees with specialized expertise.

Page Ref: 204-205

Learning Outcome: Discuss the factors that influence decisions about organizational structure

Difficulty: Difficult

Objective: 7.3 What do organization charts show, and what three skills do managers need?

Classification: Application

AACSB: Reflective Thinking Skills

79) As a supervisory manager, Zelma supervises 12 employees in her San Diego office. Her team members work well together, executing the day-to-day tasks of the organization in a highly effective manner. But recently, a transfer from a different regional office joined the team. The new employee followed very different operational procedures in her home office before coming to San Diego. What skills should Zelma utilize to help the new employee have a smooth transition in learning new operational procedures?

  1. A) conceptual skills
  2. B) technical skills
  3. C) time-management skills
  4. D) human skills
  5. E) decision-making skills

Answer: D

Explanation: D) Human skills allow Zelma to communicate company operating systems and culture to help the new employee feel comfortable and to fit in with current employees.

Page Ref: 205

Learning Outcome: Describe the skills and functions of management

Difficulty: Moderate

Objective: 7.3 What do organization charts show, and what three skills do managers need?

Classification: Application

AACSB: Communication Abilities

80) What type of skills refer to the ability to think abstractly, picture an organization as a whole, and understand how the parts work together?

  1. A) conceptual skills
  2. B) technical skills
  3. C) time-management skills
  4. D) group think skills
  5. E) decision-making skills

Answer: A

Explanation: A) Conceptual skills require the ability to think analytically. They are especially important for top managers, who frequently have to deal with ambiguous situations with far-reaching consequences.

Page Ref: 206

Learning Outcome: Describe the skills and functions of management

Difficulty: Moderate

Objective: 7.3 What do organization charts show, and what three skills do managers need?

Classification: Concept

81) The executive director of Big Deals Inc. wants to implement a new information system in the accounting department to increase the company’s effectiveness. She knows that the system will benefit the accounting department, but she also carefully considers how implementation of the new system might affect other departments within the company. What skills is the executive director utilizing?

  1. A) conceptual skills
  2. B) technical skills
  3. C) time-management skills
  4. D) human skills
  5. E) decision-making skills

Answer: A

Explanation: A) Great conceptual skills allow someone to understand the relationships between the different parts of the organization.

Page Ref: 206

Learning Outcome: Describe the skills and functions of management

Difficulty: Moderate

Objective: 7.3 What do organization charts show, and what three skills do managers need?

Classification: Application

82) Which of the following is NOT an important skill for a successful manager?

  1. A) human skills
  2. B) accounting skills
  3. C) decision-making skills
  4. D) technical skills
  5. E) conceptual skills

Answer: B

Explanation: B) Accounting is a business skill, but not an important skill for a successful manager who is not in an accounting function.

Page Ref: 206

Learning Outcome: Describe the skills and functions of management

Difficulty: Moderate

Objective: 7.3 What do organization charts show, and what three skills do managers need?

Classification: Concept

83) Marni, the top manager of a large facilities management company, believes strongly in a democratic management style. Earlier in the week, one of Marni’s employees asked a department manager whether she could schedule a meeting with Marni to discuss possibly implementing a new workflow system with which the employee already has years of experience. What is most likely Marni’s reaction when the department manager asks her about the meeting?

  1. A) Marni declines a meeting and encourages the employee to implement the workflow process and report back with the results in the next few weeks.
  2. B) Marni declines a meeting, reinforcing that she is too busy to deal with small requests from nonmanagerial employees.
  3. C) Marni schedules a meeting because she is the person in authority and should be making the major decisions for the organization.
  4. D) Marni schedules a meeting but has her assistant facilitate the meeting between the department manager and the employee.
  5. E) Marni documents in the employee’s performance review that she shows initiative but does not encourage her to implement the new system.

Answer: A

Explanation: A) Managers who believe in democratic management encourage all employees to feel empowered to make decisions.

Page Ref: 208

Learning Outcome: Discuss the factors that affect motivation and behavior in the workplace

Difficulty: Difficult

Objective: 7.3 What do organization charts show, and what three skills do managers need?

Classification: Application

AACSB: Reflective Thinking Skills

84) Effective leadership styles vary according to circumstances and the organization. Compared to other leadership styles, however, participative leadership more consistently leads to which of the following?

  1. A) enhanced job satisfaction of employees
  2. B) employee confusion about company goals
  3. C) companies with a militaristic orientation
  4. D) increased employee effectiveness
  5. E) managers who give employees freedoms

Answer: A

Explanation: A) In a participative leadership environment, employees’ opinions are sought and listened to. Managers in this type of organization tend to have good communication skills and are more empathetic.

Page Ref: 208

Learning Outcome: Summarize the major theories of and approaches to leadership

Difficulty: Moderate

Objective: 7.4 How do leaders and managers differ, and what are the different types of leaders?

Classification: Concept

85) Which of the following styles of leadership benefits the group by valuing people’s inputs and obtaining their commitment through participation?

  1. A) affiliative
  2. B) autocratic
  3. C) free-rein
  4. D) democratic
  5. E) visionary

Answer: D

Explanation: D) Democratic, or participative, leadership requires a high level of communication between the leader and the led.

Page Ref: 208

Learning Outcome: Summarize the major theories of and approaches to leadership

Difficulty: Moderate

Objective: 7.4 How do leaders and managers differ, and what are the different types of leaders?

Classification: Concept

AACSB: Communication Abilities

86) Under which of the following conditions would an autocratic style of leadership be considered most appropriate?

  1. A) when a company wants to kick-start an urgent turnaround or is dealing with a problem employee
  2. B) when a company wants to help competent, motivated employees improve performance by building long-term capabilities
  3. C) when a company wants to build buy-in or consensus or wants to get valuable input from employees
  4. D) when a company wants to enhance job satisfaction among its employees
  5. E) when a company wants its employees to discover their own ways of achieving objectives

Answer: A

Explanation: A) Some situations require a “command and control” type of direction from leadership so that there is no question of who has the authority or what the path forward is. This style may also be warranted when a change in focus needs to take place quickly and without discussion.

Page Ref: 208

Learning Outcome: Summarize the major theories of and approaches to leadership

Difficulty: Moderate

Objective: 7.4 How do leaders and managers differ, and what are the different types of leaders?

Classification: Concept

87) Which type of employees would a free-rein-organized company most likely want to hire?

  1. A) workers who need constant guidance
  2. B) workers who do not think for themselves
  3. C) workers who are self-motivated
  4. D) workers who are not good communicators
  5. E) workers who have been trained to think alike

Answer: C

Explanation: C) A free-rein, or laissez-faire, leadership style works best with workers who are skilled enough to think for and guide themselves.

Page Ref: 208

Learning Outcome: Summarize the major theories of and approaches to leadership

Difficulty: Moderate

Objective: 7.4 How do leaders and managers differ, and what are the different types of leaders?

Classification: Concept

+
-
Only 0 units of this product remain

You might also be interested in